Крок 2 - Медицина 2015 (буклет)

1 / 200
Хворому 20-ти років, що доставлений влітку з вулиці із кровотечею із плечової артерії, при наданні першої медичної допомоги з метою тимчасової зупинки кровотечі було накладено джгут. Вкажіть максимальну експозицію джгута: A 20-year-old patient brought in from the street in the summer with bleeding from the brachial artery, a tourniquet was applied during first aid to temporarily stop the bleeding. Specify the maximum harness exposure:

30 хвилин 30 minutes

60 хвилин 60 minutes

15 хвилин 15 minutes

180 хвилин 180 minutes

120 хвилин 120 minutes

2 / 200
У хворої після підняття важкої сумки раптово з’явився гострий біль у попереку. Рухи в хребті обмежені. Не викликається ахіловий рефлекс зліва, з’явилась анестезія больової чутливості на зовнішній поверхні лівої гомілки. Яке захворювання запідозрите? After lifting a heavy bag, the patient suddenly developed a sharp pain in the lower back. Movements in the spine are limited. The Achilles reflex on the left side is not evoked, pain sensitivity anesthesia appeared on the outer surface of the left lower leg. What disease do you suspect?

Люмбаго Lumbago

Попереково-крижовий радикуліт Lumbosacral sciatica

Люмбалгія Lumbalgia

Неврит стегнового нерва Femoral neuritis

Спінальний арахноїдит Spinal arachnoiditis

3 / 200
У хворої з вираженим менінгеальним синдромом, петехіальними висипаннями на шкірі, ознобом, температурою тіла 39°С, запальними змінами в периферичній крові та нейтрофільним плеоцитозом в лікворі діагностовано гнійний менінгіт. Який із наявних синдромів у хворої має вирішальне значення для постановки діагнозу менінгіту? A patient with severe meningeal syndrome, petechial rashes on the skin, chills, body temperature of 39°С, inflammatory changes in the peripheral blood and neutrophilic pleocytosis in the cerebrospinal fluid was diagnosed with purulent meningitis Which of the syndromes present in the patient is of decisive importance for the diagnosis of meningitis?

Нейтрофільний плеоцитоз Neutrophil pleocytosis

Підвищення температури тіла Increasing body temperature

Петехіальні висипання на шкірі Petechial rash on the skin

Менінгеальний синдром Meningeal syndrome

Запальні зміни в крові Inflammatory changes in the blood

4 / 200
У 31-річного хворого з’явився різкий озноб, температура 39°C, що трималась 9 годин і раптово знизилась до нормальної з сильним потовиділенням. Наступного дня температура 36,8°C, Ps- 70/хв. Живіт м’який, не болючий. Печінка +2 см, селезінка +3 см. Рік тому повернувся з Південної Азії, де було декілька нападів гарячки. Яке дослідження найкраще провести для швидкого підтвердження діагнозу? A 31-year-old patient developed a sharp chill, a temperature of 39°C, which lasted for 9 hours and suddenly dropped to normal with profuse sweating. The next day, the temperature was 36 .8°C, Ps- 70/min. Abdomen soft, not painful. Liver +2 cm, spleen +3 cm. Returned from South Asia a year ago, where there were several attacks of fever. What investigation is best to do to quickly confirm the diagnosis ?

Дослідження товстої краплі крові на малярію Thick blood drop test for malaria

Дослідження крові на стерильність Blood test for sterility

Дослідження крові на anti-HEV IgM Blood test for anti-HEV IgM

Бактеріоскопія товстої краплі крові на лептоспіроз Bacterioscopy of a thick drop of blood for leptospirosis

Бактеріоскопія товстої краплі крові на менінгокок Bacterioscopy of a thick drop of blood for meningococcus

5 / 200
У хворого 37-ми років через 2 доби після дорізу великої рогатої худоби з’явилась пляма на руці, яка за добу перетворилася на пустулу з чорним дном, безболісну під час дотику, з вінцем дочірніх везикул по периферії. На руці та плечі безболісний набряк. Підвищилась температура тіла до 39° Ps- 100/хв., АТ- 95/60 мм рт.ст., ЧД- 30/хв. Який діагноз найбільш імовірний? A 37-year-old patient developed a spot on his arm 2 days after slaughtering cattle, which turned into a pustule with a black bottom within a day, painless under time of touch, with a crown of daughter vesicles on the periphery. Painless edema on the arm and shoulder. Body temperature rose to 39° Ps- 100/min., BP- 95/60 mm Hg, BH- 30/min. What diagnosis is the most likely?

Бруцельоз Brucellosis

Туляремія Tularemia

Чума Plague

Оперізуючий герпес Herpes zoster

Сибірка Anthrax

6 / 200
Юнак 17-ти років на призовній комісії поскаржився на шум у вухах, який підсилюється при фізичному навантаженні. АТ-150/30 мм рт.ст., вислуховувається діастолі-чний шум над аортою. На оглядовій рентгенограмі тінь серця аортальної конфігурації, збільшені дуга аорти та лівий шлуночок. Легеневий малюнок не змінений. Пульсація аорти підсилена. Який найбільш імовірний діагноз? A 17-year-old boy complained to the draft board about tinnitus, which worsens with physical exertion. BP-150/30 mm Hg, diastole is heard - noise above the aorta. On the X-ray examination, the shadow of the heart of the aortic configuration, the aortic arch and the left ventricle are enlarged. The pulmonary pattern is unchanged. The pulsation of the aorta is increased. What is the most likely diagnosis?

Аортальна недостатність Aortic insufficiency

Гіпертонічна хвороба Hypertensive disease

Коарктація аорти Coarctation of the aorta

Атеросклероз аорти Atherosclerosis of the aorta

Декстрапозиція аорти Dextraposition of the aorta

7 / 200
Дитині 5 років. Знаходиться на диспансерному обліку у кардіолога з приводу вродженої вади серця. Мати звернулась до лікаря зі скаргами на появу задишки при фізичному навантаженні та у спокої, нестійкі набряки гомілок і стоп, які під кінець дня посилюються і зникають після нічного сну. З анамнезу відомо, що 3 тижні назад дитина перехворіла на ангіну. Вкажіть походження периферичних набряків: The child is 5 years old. He is under the dispensary record of a cardiologist due to a congenital heart defect. The mother turned to the doctor with complaints of shortness of breath during physical exertion and at rest, unstable swelling of the lower legs and feet, which intensifies at the end of the day and disappears after a night's sleep. From the anamnesis, it is known that the child fell ill with angina 3 weeks ago. Specify the origin of the peripheral edema:

Цирротичне Cirrhotic

Ниркове Renal

Внаслідок тромбофлебіту Due to thrombophlebitis

Ортостатичне Orthostatic

Серцеве Cardiac

8 / 200
Хворий 42-х років скаржиться на постійний біль в епігастральній ділянці з іррадіацією в спину, нудоту, блювання, печію. Хворіє на виразкову хворобу duodenum протягом 20-ти років. Об’єктивно: пониженого харчування, язик обкладений, вологий. Живіт напружений, болючий під час пальпації в пілородуоденальній зоні. Позитивний симптом Образцова. Субфебрилітет. У крові: лейк.- 10,0 • 109 /л, ШЗЕ - 24 мм/год. Рентгенологічно: в цибулині duodenum занурена тришарова ніша 0,7x1,0 см. Реакція Грегерсена негативна. Яке ускладнення має місце в даному випадку? A 42-year-old patient complains of constant pain in the epigastric region with radiation to the back, nausea, vomiting, heartburn. He has been suffering from duodenal ulcer disease for 20 years . Objectively: undernourished, tongue coated, moist. Abdomen is tense, painful during palpation in the pyloroduodenal zone. Positive symptom of Obraztsov. Subfebrile. In blood: leuk.- 10.0 • 109 /l, SZE - 24 mm/h X-ray: a three-layer niche 0.7x1.0 cm is immersed in the bulb of the duodenum. Gregersen's reaction is negative. What complication occurs in this case?

Стеноз воротаря Stenosis of the goalkeeper

Кровотеча Bleeding

Перфорація Perforation

Пенетрація Penetration

Малігнізація Malignancy

9 / 200
Чоловік 55-ти років скаржиться на загальну слабкість, зменшення сечовиділення, шкірний свербіж. Протягом 15-ти років страждає на хронічний пієлонефрит. Об’єктивно: шкірні покриви сухі, з жовтуватим відтінком. Ps- 80/хв., ритмічний, АТ- 100/70 мм рт.ст. При аускультації тони серця глухі, вислуховується шум тертя перикарду. Креатинін крові -1,1 ммоль/л, клубочкова фільтрація 5 мл/хв. Яке лікування показано хворому? A 55-year-old man complains of general weakness, decreased urination, skin itching. He has been suffering from chronic pyelonephritis for 15 years. Objectively: the skin is dry , with a yellowish tinge. Ps- 80/min., rhythmic, BP- 100/70 mm Hg. On auscultation, heart sounds are dull, pericardial friction noise is heard. Blood creatinine - 1.1 mmol/l, glomerular filtration 5 ml/ min. What treatment is indicated for the patient?

Неогемодез Neohemodez

Плазмаферез Plasmapheresis

Гемодіаліз Hemodialysis

Сечогінні Diuretics

Ентеросорбент Enterosorbent

10 / 200
Народилася дитина від фізіологічних пологів у строк. На другу добу у дитини з’явилась іктерічність шкіри та слизових оболонок. Непрямий білірубін дорівнював 136 мкмоль/л. У матері група крові 0(1) Rh- , у дитини - A(II)Rh+ . Який механізм виникнення жовтяниці? A child was born from a physiological delivery at term. On the second day, the child developed jaundice of the skin and mucous membranes. Indirect bilirubin was equal to 136 μmol/l. The mother's group blood is 0(1) Rh-, the child has A(II)Rh+. What is the mechanism of jaundice?

Порушення відтоку жовчі Bile outflow disorder

Гепатит Hepatitis

Гемоліз еритроцитів Hemolysis of erythrocytes

Порушення обміну білірубіну Bilirubin metabolism disorder

Холестаз Cholestasis

11 / 200
У хворої 4 доби тому з’явився висип на шкірі, що супроводжується свербінням. Турбує здуття живота, тупий біль у правому підребер’ї, закреп. За день до цього з’їла копчене м’ясо. Раніше схожі реакції з’являлися після вживання значної кількості томатів, полуниці, шоколаду. Об’єктивно: на шкірі обличчя, тулуба, кінцівок уртикарний висип. Рівень загального IgE у нормі. Який найбільш імовірний діагноз? The patient developed a skin rash accompanied by itching 4 days ago. She is bothered by abdominal distension, dull pain in the right hypochondrium, constipation. The day before she ate smoked meat. Previously, similar reactions appeared after eating a large amount of tomatoes, strawberries, chocolate. Objectively: an urticarial rash on the skin of the face, trunk, limbs. The level of total IgE is normal. What is the most likely diagnosis?

Атопічний дерматит Atopic dermatitis

Хронічна кропив’янка Chronic urticaria

Ідіосинкразія Idiosyncrasy

Псевдоалергія Pseudoallergy

Харчова алергія Food allergy

12 / 200
Під час бесіди з лікарем хвора розповіла: 'Кожну ніч при засинанні до мене приходить 'вісник смерті', я проходжу повз неї та роблю вигляд, що її не бачу. Потім вона підкрадається до мого ліжка з сокирою: сокира виблискує, а її очі світяться люттю. Таке відбувається багато разів за ніч. Визначте стан хворого: During the conversation with the doctor, the patient said: 'Every night when I fall asleep, a 'harbinger of death' comes to me, I walk past her and pretend I don't see her . Then she sneaks up to my bed with an axe: the ax glitters and her eyes glow with rage. This happens many times a night. Determine the patient's condition:

Гіпнагогічні галюцинації Hypnagogic hallucinations

Псевдогалюцінації Pseudohallucinations

Насильницьки спогади Violent memories

Ментизм Mentism

Патологічний афект Pathological affect

13 / 200
Жінці 57-ми років з ожирінням III ст. два місяці тому встановлено діагноз цукрового діабету. Ендокринологом хворій було рекомендовано субкалорійну дієту та дозовані фізичні навантаження. Рівень глікемії натще 9,2 ммоль/л. Який цукрознижуючий препарат можна рекомендувати хворій? A 57-year-old woman with third-degree obesity was diagnosed with diabetes two months ago. The endocrinologist recommended the patient a low-calorie diet and limited physical activity. The fasting blood glucose level was 9 .2 mmol/l. What hypoglycemic drug can be recommended to the patient?

Репаглінід Repaglinide

Гліквідон Glyquidon

Гліпізид Glipizide

Метформін Metformin

Глібенкламід Glibenclamide

14 / 200
Хвора 73-х років скаржиться на слабкість, сонливість, мерзлякуватість, різке погіршення пам’яті, випадіння волосся, закрепи, набряки. Об’єктивно: нормального харчування. Шкіра суха, жовтувата. Обличчя та кінцівки набряклі, при натисканні ямки не залишається. Серцеві тони приглушені, брадикардія. Розміри серця розширені. Об’єм щитоподібної залози зменшений. У крові: НЬ- 85 г/л, холестерин - 8,5 ммоль/л, ТТГ 20,5 мкмоль/л. Який попередній діагноз? A 73-year-old patient complains of weakness, drowsiness, chills, sharp deterioration of memory, hair loss, constipation, swelling. Objectively: normal diet. The skin is dry, yellowish. The face and limbs are swollen, when pressed there is no fossa. The heart sounds are muffled, bradycardia. The heart is enlarged. The volume of the thyroid gland is reduced. In the blood: Hb - 85 g/l, cholesterol - 8.5 mmol/ l, TSH 20.5 μmol/l. What is the previous diagnosis?

Кардіосклероз, серцева недостатність Cardiosclerosis, heart failure

Хронічний гепатит Chronic hepatitis

Ниркова недостатність Kidney failure

Гіпотиреоз Hypothyroidism

Виражений атеросклероз мозкових судин Marked atherosclerosis of cerebral vessels

15 / 200
В ревматологічне відділення поступила хвора 20-ти років з явищами поліартриту. На шкірі обличчя - почервоніння у вигляді 'метелика'. У сечі - білок 4,8 г/л. Лікар запідозрив наявність у хворої системного червоного вовчаку. Який додатковий метод дослідження є найбільш інформативним для верифікації діагнозу? A 20-year-old patient with symptoms of polyarthritis was admitted to the rheumatology department. Redness in the form of a 'butterfly' on the skin of the face. Protein 4.8 g/ l. The doctor suspected the presence of systemic lupus erythematosus in the patient. What additional research method is the most informative for verifying the diagnosis?

Аналіз крові на антинуклеарні антитіла Blood analysis for antinuclear antibodies

Загальний аналіз крові General blood test

Шунологічне дослідження крові Chunological examination of blood

Аналіз крові на ревматоїдний фактор Blood analysis for rheumatoid factor

Аналіз крові на LE-клітини Blood analysis for LE cells

16 / 200
Хворий 19-ти років скаржиться на свербіж у міжпальцевих складках кистей, шкіри живота, який посилюється ввечері. Хворіє протягом 2 тижнів. Об’єктивно: в міжпальцевих складках кистей, на шкірі живота велика кількість розміщених попарно папуловезикул, розчухів, геморагічних кірочок. У зскрібку виявлено Sarcoptes scabiei hominis. Який найбільш імовірний діагноз? A 19-year-old patient complains of itching in the interdigital folds of the hands, abdominal skin, which worsens in the evening. He has been sick for 2 weeks. Objectively: in the interdigital folds of the hands , on the skin of the abdomen, a large number of paired papulovesicles, abrasions, hemorrhagic crusts. Sarcoptes scabiei hominis was found in the scraping. What is the most likely diagnosis?

Червоний плоский лишай Red lichen planus

Мікробна екзема Microbial Eczema

Контактний дерматит Contact dermatitis

Короста Scabies

Нейродерміт Neurodermatitis

17 / 200
У хлопчика у віці 1 місяць виникає блювання фонтаном після кожного годування. Блювотні маси являють собою звурджене молоко і перевищують за обсягом попереднє годування. Дитина за перший місяць набрала у вазі 200 г. Сечовиділення рідкі, випорожнення мізерні, нерегулярні. Який метод обстеження ви призначите для верифікації діагнозу? A 1-month-old boy vomits in a fountain after each feeding. The vomited masses are curdled milk and exceed the volume of the previous feeding. The child gained weight in the first month 200 g. Urine secretion is liquid, stools are scanty, irregular. What examination method will you prescribe to verify the diagnosis?

Рентгенографія черевної порожнини X-ray of the abdominal cavity

Біохімічне дослідження Biochemical research

Гастрофіброскопічне дослідження Gastrofibroscopic examination

Копрологічне дослідження Coprological research

Ультразвукове дослідження Ultrasound examination

18 / 200
Хвора 39-ти років впродовж 10-ти років хворіє на цукровий діабет. Останній рік відмічає похолодання пальців ніг, біль та відчуття оніміння. Об’єктивно: шкіра нижніх кінцівок суха, витончена, холодна на дотик, пульсація на стегновій і підколінній артеріях збережена. Який найбільш імовірний діагноз? A 39-year-old patient has been suffering from diabetes for 10 years. The last year has been marked by coldness of the toes, pain and a feeling of numbness. Objectively: the skin of the lower the extremities are dry, thin, cold to the touch, pulsation in the femoral and popliteal arteries is preserved. What is the most likely diagnosis?

Діабетична макроангіопатія судин нижніх кінцівок Diabetic macroangiopathy of the vessels of the lower extremities

Облітеруючий атеросклероз судин нижніх кінцівок Obliterating atherosclerosis of the vessels of the lower extremities

Хвороба Рейно Raynaud's disease

Діабетична мікроангіопатія судин нижніх кінцівок Diabetic microangiopathy of the vessels of the lower extremities

Облітеруючий ендартеріїт судин нижніх кінцівок Obliterating endarteritis of the vessels of the lower extremities

19 / 200
У хворого 48-ми років після вживання 700 мл горілки та великої кількості їжі з’явилися нестримне блювання, інтенсивний біль за грудниною та в епігастральній ділянці з іррадіацією у хребет, задишка. Втратив свідомість. Об’єктивно: ЧД- 28/хв. Рs- 54/хв. АТ80/50 мм рт.ст. Підшкірна крепітація в ділянці шиї, ціаноз обличчя. Тони серця послаблені. Дихання везикулярне з обох боків. Напруження м’язів у епігастральній ділянці. Чим може бути обумовлений стан хворого? A 48-year-old patient, after consuming 700 ml of vodka and a large amount of food, developed uncontrollable vomiting, intense pain behind the sternum and in the epigastric area with radiation to the spine , shortness of breath. Lost consciousness. Objectively: BH- 28/min. Ps- 54/min. BP80/50 mm Hg. Subcutaneous crepitation in the neck area, cyanosis of the face. Heart tones are weakened. Breathing is vesicular on both sides. Tension muscles in the epigastric area. What could be the cause of the patient's condition?

Тромбоемболія легеневої артерії Thromboembolism of the pulmonary artery

Розшаровуюча аневризма аорти Dissecting aortic aneurysm

Пневмоторакс Pneumothorax

Гострий інфаркт міокарда Acute myocardial infarction

Розрив стравоходу Rupture of esophagus

20 / 200
Хворий 24-х років звернувся до лікаря зі скаргами на збільшення підщелепних лімфовузлів. Об’єктивно: збільшені підщелепні, пахвові та пахвинні лімфатичні вузли. На рентгенограмі органів грудної клітки -збільшені лімфовузли середостіння. У крові: ер.- 3,4 • 1012/л, НЬ-100 г/л, КП- 0,88, тр.-190 • 109 /л, лейк.- 7, 5 • 109 /л, е.- 8%, п.- 2%, с.- 67%, лімф.- 23%, ШОЕ- 22 мм/год. Яке дослідження показане для верифікації причини лімфаденопатії? A 24-year-old patient turned to the doctor with complaints of enlarged submandibular lymph nodes. Objectively: enlarged submandibular, axillary and inguinal lymph nodes. On chest X-ray - enlarged lymph nodes of the mediastinum. In the blood: er.- 3.4 • 1012/l, Hb-100 g/l, KP- 0.88, tr.-190 • 109 /l, leuk.- 7.5 • 109 / l, e.- 8%, p.- 2%, s.- 67%, lymph.- 23%, SOE- 22 mm/h. What research is indicated to verify the cause of lymphadenopathy?

Відкрита біопсія лімфатичних вузлів Open biopsy of lymph nodes

Пункційна біопсія лімфатичних вузлів Puncture biopsy of lymph nodes

Ультразвукове дослідження органів черевної порожнини Ultrasound examination of abdominal organs

Томографія середостіння Tomography of the mediastinum

Стернальна пункція Sternal puncture

21 / 200
Хворий 32-х років звернувся зі скаргами на перебої в роботі серця, запаморочення, задишку при фізичному навантаженні. До цього часу не хворів. Об’єктивно: Рs-74/хв., ритмічний. АТ- 130/80 мм рт.ст. При аускультації: перший тон нормальної звучності, систолічний шум над аортою. На ЕКГ: гіпертрофія лівого шлуночку, ознаки порушення реполяризації в І, V5, V6 відведеннях. ЕхоКГ: міжшлуночкова перегородка 2 см. Який найбільш вірогідний діагноз? A 32-year-old patient complained of heart failure, dizziness, shortness of breath during physical exertion. Until then, he had not been sick. Objectively: Ps- 74/min., rhythmic. BP - 130/80 mm Hg. On auscultation: the first tone of normal sonority, systolic murmur over the aorta. On the ECG: hypertrophy of the left ventricle, signs of repolarization disorders in leads I, V5, V6. Echocardiography: interventricular septum 2 cm. What is the most likely diagnosis?

Інфаркт міокарду Myocardial infarction

Стеноз гирла аорти Stenosis of the mouth of the aorta

Гіпертрофічна кардіоміопатія Hypertrophic cardiomyopathy

Гіпертонічна хвороба Hypertensive disease

Коарктація аорти Coarctation of the aorta

22 / 200
У дівчинки 6-ти років на фоні ГРВІ з’явилися зміни в аналізі сечі: сліди білку, лейкоцити 30-40 в п/з, еритроцити (свіжі) 10-12 в п/з. АТ- 100/60 мм рт.ст. Який з наведених діагнозів найбільш імовірний? A 6-year-old girl had changes in the urine analysis against the background of SARS: traces of protein, leukocytes 30-40 in p/z, erythrocytes (fresh) 10-12 in p/z. Blood pressure - 100/60 mm Hg. Which of the following diagnoses is the most probable?

Інфекція сечовивідної системи Urinary tract infection

Гострий гломерулонефрит Acute glomerulonephritis

Вульвовагініт Vulvovaginitis

Сечокам’яна хвороба Urolithiasis

Геморагічний васкуліт Hemorrhagic vasculitis

23 / 200
Клінікою НДІ професійних хвороб встановлено у робітника, який працює на збагачувальній фабриці, професійне захворювання - хронічний пиловий бронхіт. Розслідування випадку проводить комісія у складі представників: підприємства, медикосанітарної частини, територіальної СЕС, відділення Фонду соціального страхування, профспілкової організації. Представник якого закладу повинен очолити роботу комісії згідно з чинним 'Положенням про розслідування...' ? The clinic of the Scientific Research Institute of Occupational Diseases diagnosed a worker who works at an enrichment factory with an occupational disease - chronic dust bronchitis. The investigation of the case is carried out by a commission composed of representatives of: the enterprise, the medical and sanitary department , territorial SES, branch of the Social Insurance Fund, trade union organization. The representative of which institution should head the work of the commission according to the current 'Regulations on Investigations...

Територіальної СЕС Territorial SES

Фонду соціального страхування Social Insurance Fund

Медико-санітарної частини Medical and sanitary department

Підприємства Enterprises

Профспілкової організації Trade union organization

24 / 200
38-ми років у крові: НЬ- 84 г/л, ер.- 3, 5 • 1012/л, КП- 0,72, лейк.- 4,1 • 109 24. /л; ретик.- 2%, ШОЕ- 26 мм/год. Залізо сироватки крові - 9,0 ммоль/л. Було призначено лікування. На 6-й день лікування проведено дослідження периферичної крові. Які зміни показників дозволять найбільш імовірно підтвердити адекватність призначеної терапії? 38-year-old's blood: Hb- 84 g/l, er.- 3.5 • 1012/l, KP- 0.72, leuk. - 4.1 • 109 24. /l; retic.- 2%, ESR - 26 mm/h. Iron in blood serum - 9.0 mmol/l. Treatment was prescribed. On the 6th day of treatment, a study of peripheral blood was performed. What changes in indicators will most likely confirm the adequacy of the prescribed therapy?

Нормалізація кількості еритроцитів Normalization of the number of erythrocytes

Зниження швидкості зсідання еритроцитів Decreased erythrocyte sedimentation rate

Підвищення колірного показника Increasing color index

Підвищення кількості ретикулоцитів Increase in the number of reticulocytes

Нормалізація рівня гемоглобіну Normalization of hemoglobin level

25 / 200
До інфекційного відділення надійшла жінка 47-ми років зі скаргами на високу температуру, озноб, сильну слабкість, головний біль, нудоту і блювання. Стул частий, рідкий. Захворювання почалося через 12 годин після вживання у їжу омлету з качиних яєць. Який найбільш імовірний діагноз? A 47-year-old woman came to the infectious disease department with complaints of high temperature, chills, severe weakness, headache, nausea and vomiting. The stool is frequent, loose. Disease started 12 hours after eating a duck egg omelet What is the most likely diagnosis?

Ботулізм Botulism

Стафілококове харчове отруєння Staphylococcal food poisoning

Сальмонельоз Salmonellosis

Харчове отруєння, викликане V. Parahaemolyticus Food poisoning caused by V. Parahaemolyticus

Харчове отруєння, викликане B. cereus Food poisoning caused by B. cereus

26 / 200
У хворої вранці з’явилися нудота, одноразове блювання, сухість у роті. Ввечері відзначила двоїння предметів, 'туман' перед очима, поперхування під час ковтання їжі. Об’єктивно: температура 36,4°С, птоз, мідріаз, анізокорія, відсутність блювотного і ковтального рефлексів, сухість слизових. З боку внутрішніх органів відхилень не виявлено. Яке захворювання найбільш імовірне? In the morning, the patient developed nausea, one-time vomiting, dry mouth. In the evening, she noticed double vision of objects, 'fog' in front of her eyes, and drooling when swallowing food. About objectively: temperature 36.4°С, ptosis, mydriasis, anisocoria, absence of vomiting and swallowing reflexes, dryness of mucous membranes. No abnormalities were detected on the part of internal organs. What disease is most probable?

Стовбуровий енцефаліт Stem encephalitis

Ботулізм Botulism

Гостре порушення мозкового кровообігу Acute violation of cerebral circulation

Вірусний менінгоенцефаліт Viral meningoencephalitis

Туберкульозний менінгоенцефаліт Tuberculous meningoencephalitis

27 / 200
У хворого 16-ти років, що страждає на виражену кровоточивість при невеликих порізах, ранках, виникло питання про необхідність екстракції коренів зубів. Об’єктивно: збільшення в об’ємі правого колінного суглоба, обмеження рухомості. Інших змін немає. У крові: тенденція до анемії (Hb-120 г/л). Чим необхідно здійснювати профілактику кровоточивості перед втручанням стоматолога? A 16-year-old patient suffering from pronounced bleeding from small cuts and wounds, the question arose about the need for tooth root extraction. Objectively: an increase in 'swelling of the right knee joint, limitation of mobility. There are no other changes. In the blood: a tendency to anemia (Hb-120 g/l). What should be done to prevent bleeding before the intervention of the dentist?

Кріопреципітат Cryoprecipitate

Епсилон-амінокапронова кислота Epsilon-aminocaproic acid

Суха плазма крові Dry blood plasma

Фібриноген Fibrinogen

Вливання хлористого кальцію Infusion of calcium chloride

28 / 200
Хворий 40-ка років скаржиться на болі у лівому кульшовому суглобі, що посилюються під час ходи, підвищення температури до 37, 7°С впродовж 2-х місяців. В анамнезі: вогнищевий туберкульоз легень. На рентгенограмі лівого кульшового суглоба визначається деструкція суміжних поверхонь головки стегнової кістки і даху вертлюгової западини, звуження суглобової щілини. Проба Манту з 2 ТО - папула 14 мм. Який з перелічених діагнозів найбільш імовірний? A 40-year-old patient complains of pain in the left hip joint that worsens when walking, temperature rise to 37.7°C for 2 months. In the anamnesis: focal tuberculosis of the lungs. An X-ray of the left hip joint reveals destruction of the adjacent surfaces of the femoral head and the roof of the acetabulum, narrowing of the joint space. Mantoux test with 2 TO - papule 14 mm. Which of the listed diagnoses is the most probable?

Туберкульозний коксит зліва Tuberculous coxitis on the left

Артрозо-артрит лівого кульшового суглоба Arthrozo-arthritis of the left hip joint

Ревматоїдний артрит Rheumatoid arthritis

Саркома лівого кульшового суглоба Sarcoma of the left hip joint

Ревматичний артрит Rheumatic arthritis

29 / 200
У хворого 30-ти років, курця, під час нападу кашлю з’явився різкий біль у правій половині грудної клітки, задишка у спокої, різка загальна слабкість. Запідозрено пневмоторакс. Над правою половиною грудної клітки тимпаніт, дихання відсутнє. ЧД-36/хв., Рs- 120/хв., ритмічний, АТ- 90/60 мм рт.ст. У патогенезі даного стану провідна роль належить такому чиннику: A 30-year-old patient, a smoker, during a coughing attack developed sharp pain in the right side of the chest, shortness of breath at rest, and sharp general weakness. It is suspected pneumothorax. Tympanitis over the right half of the chest, no breathing. BH-36/min., Ps- 120/min., rhythmic, BP- 90/60 mm Hg. In the pathogenesis of this condition, the leading role belongs to the following factor:

Тромбоз дрібних коронарних артерій Thrombosis of small coronary arteries

Зниження скоротливої здатності лівого шлуночка Decreased contractility of the left ventricle

Підвищення тиску в малому колі кровообігу Increased pressure in the small circle of blood circulation

Різке підвищення внутрішньоплеврального тиску Sudden increase in intrapleural pressure

Механічна закупорка гілок легеневої артерії Mechanical blockage of pulmonary artery branches

30 / 200
Хвора 34-х років захворіла гостро. Скарги на слабкість, млявість, відчуття стискання у грудях, утруднене дихання. У хворої парадоксальний пульс, переповнення вен шиї. Який попередній діагноз? A 34-year-old patient became acutely ill. Complaints of weakness, lethargy, a feeling of tightness in the chest, difficulty breathing. The patient has a paradoxical pulse, overflowing neck veins. What is the previous diagnosis?

Гострий перикардит Acute pericarditis

Інфаркт міокарда Myocardial infarction

Туберкульозний лобіт Tuberculosis lobit

Гостра пневмонія Acute pneumonia

Пухлина легені Lung tumor

31 / 200
Районний центр не каналізований, нечистоти збираються у вигрібних ямах дворових вбиралень. Для знешкодження нечистот районного центра слід застосувати: The district center is not sewered, sewage collects in cesspools of yard latrines. To neutralize the sewage of the district center, you should use:

Біологічні ставки Biological rates

Землеробні поля зрошення Irrigation agricultural fields

Поля асинізації Asynization fields

Поля зрошення Irrigated fields

Поля фільтрації Filter fields

32 / 200
Пацієнт 40-ка років, гомосексуаліст, звернувся зі скаргами на тривалу гарячку, нічну пітливість, різке зниження маси тіла, біль у м’язах, горлі, суглобах, рецидивуючу діарею. Об’єктивно: генералізована лімфаденопатія. Яке захворювання слід запідозрити? A patient in his 40s, a homosexual, complained of prolonged fever, night sweats, a sharp decrease in body weight, pain in the muscles, throat, joints, recurrent diarrhea. Objectively: generalized lymphadenopathy. What disease should be suspected?

СНІД AIDS

Лімфолейкоз Lympholeukosis

Туберкульоз Tuberculosis

Лімфогрануломатоз Lymphogranulomatosis

Сепсис Sepsis

33 / 200
Хворий 42-х років раптово, на фоні повного здоров’я, відчув біль по типу 'удару по голові', після чого розвилось блювання, втратив свідомість, генералізовані судоми. Об’єктивно: кома І, ригідність потиличних м’язів, ліва зіниця більше у розмірі ніж права, праві кінцівки ротовані, м’язовий тонус значно зменшений. Який найбільш імовірний діагноз? A 42-year-old patient suddenly, against the background of complete health, felt pain like a 'blow to the head', after which vomiting developed, he lost consciousness, generalized convulsions. Objectively: coma I, rigidity of the occipital muscles, the left pupil is larger than the right, the right limbs are rotated, the muscle tone is significantly reduced. What is the most likely diagnosis?

Епілепсія Epilepsy

Ішемічний інсульт Ischemic stroke

Розрив аневризми головного мозку Rupture of brain aneurysm

Менінгіт Meningitis

Внутрішньочерепна гематома Intracranial hematoma

34 / 200
Повторнонароджуюча 26-ти років прибула в пологове відділення у зв’язку з вагітністю 40 тижнів та початком пологової діяльності. 2 години тому відійшли навколоплідні води. Положення плоду повздовжнє, головне передлежання. ОЖ- 100 см, ВДМ- 42 см. Перейми через 4-5 хвилин, по 25 секунд. При внутрішньому акушерському обстеженні: шийка матки згладжена, відкриття 4 см. Плідного міхура немає. Головка плоду притиснута до входу в малий таз. Яке ускладнення виникло в пологах? A 26-year-old woman who gave birth again came to the maternity ward due to her 40-week pregnancy and the beginning of labor. 2 hours ago, the amniotic fluid broke. The position of the fetus is longitudinal, main presentation. OZ - 100 cm, VDM - 42 cm. Take after 4-5 minutes, 25 seconds each. During internal obstetric examination: the cervix is smooth, the opening is 4 cm. There is no fetal bladder. The head of the fetus is pressed against the entrance to the pelvis. What complication occurred during childbirth?

Дискоординована пологова діяльність Discoordinated birth activity

Вторинна слабкість пологової діяльності Secondary weakness of labor activity

Передчасне вилиття навколоплідних вод Premature discharge of amniotic fluid

Первинна слабкість пологової діяльності Primary weakness of labor activity

Клінічно вузький таз Clinically narrow pelvis

35 / 200
Хвора 35-ти років скаржиться на біль та ранкову скутість суглобів кистей рук та скроневонижньощелепних суглобів тривалістю більше 30 хвилин. Хворіє 2 роки. Об’єктивно: набряк проксимальних міжфалангових суглобів кистей та обмеженість руху в них. Яке дослідження необхідно провести? A 35-year-old patient complains of pain and morning stiffness in the joints of the hands and temporomandibular joints lasting more than 30 minutes. She has been ill for 2 years. Objectively: swelling of the proximal interphalangeal hand joints and limited movement in them. What research should be conducted?

Протеїнограма Proteinogram

Реакція Ваале-Роузе Waale-Rouse reaction

Загальний аналіз крові General blood test

Імунограма Immunogram

Рентгенографія кистей рук X-ray of hands

36 / 200
Хворий 68-ми років, службовець. Близько двох років тому почалися порушення пам’яті: став забудькуватим в роботі і побуті, не справлявся із службовими обов’язками. Поступово став забувати імена рідних, назви предметів, став безпорадним, порушилася мова. Втратив навички письма, читання, догляду за собою. При комп’ютернотомографічному дослідженні: атрофія кори великих півкуль мозку. Який діагноз є найбільш вірогідним? The patient is 68 years old, an employee. About two years ago, memory disorders began: he became forgetful at work and in everyday life, could not cope with official duties. He gradually began to forget the names of relatives, the names of objects, became helpless, his speech was impaired. He lost the skills of writing, reading, and self-care. Computed tomographic examination: atrophy of the cortex of the large hemispheres of the brain. What diagnosis is the most likely?

Сенільна деменція Senile dementia

Хвороба Піка Pick's disease

Прогресивний параліч Progressive paralysis

Атеросклеротична деменція Atherosclerotic dementia

Хвороба Альцгеймера Alzheimer's disease

37 / 200
Дівчинка 13-ти років протягом 5-ти років скаржиться на біль у правому підребер’ї, що віддає у праву лопатку, напади болю пов’язані з порушенням дієти, вони нетривалі, легко знімаються спазмолітичними засобами. Під час нападу болю пальпація живота болісна, максимально в точці проекції жовчного міхура. З найбільшою імовірністю у хворого має місце: A 13-year-old girl has been complaining of pain in the right hypochondrium for 5 years, radiating to the right shoulder blade, pain attacks are associated with a diet violation , they are short-lived, easily removed with antispasmodic agents. During an attack of pain, palpation of the abdomen is painful, maximally at the point of projection of the gallbladder. The patient is most likely to have:

Хронічний гастродуоденіт Chronic gastroduodenitis

Хронічний холецистит Chronic cholecystitis

Виразкова хвороба 12-палої кишки Duodenal ulcer

Дискінезія жовчовивідних шляхів Biliary tract dyskinesia

Хронічний панкреатит Chronic pancreatitis

38 / 200
Пацієнт 48-ми років при бужуванні стравоходу з причини рубцевої стриктури відчув різкий біль у животі. Раніше хворів на виразкову хворобу 12-ти палої кишки. При огляді: живіт різко напружений, болючий; Рs- 110/хв. Болюче ковтання слини. Блідість шкірних покривів. Який найбільш вірогідний діагноз? A 48-year-old patient felt a sharp pain in the abdomen due to a bulging esophagus due to a cicatricial stricture. He had previously suffered from duodenal ulcer disease. On examination: abdomen sharply tense, painful; Ps- 110/min. Painful swallowing of saliva. Pallor of the skin. What is the most likely diagnosis?

Защемлення діафрагмальної кили Pinching of diaphragmatic hernia

Перфорація виразки 12-ти палої кишки Perforation of duodenal ulcer

Тромбоз мезентеріальних судин Thrombosis of mesenteric vessels

Гострий інфаркт міокарда Acute myocardial infarction

Перфорація абдомінального відділу стравоходу Perforation of the abdominal part of the esophagus

39 / 200
Хвора скаржиться на нападоподібні болі в правому підребір’ї з іррадіацією в праве плече, які виникають після вживання смаженої їжі. Періодично відмічає підвищення температури тіла до субфебрильних цифр. Хворіє близько 5 років. Які обстеження необхідно провести хворій в першу чергу? The patient complains of attack-like pains in the right hypochondrium with radiation to the right shoulder, which occur after eating fried food. Periodically notices an increase in body temperature to subfebrile numbers. The patient about 5 years. What examinations should be performed on the patient first of all?

Оглядова рентгенографія Surveillance X-ray

Дуоденальне зондування Duodenal sounding

УЗД органів черевної порожнини Ultrasound of abdominal organs

Холецистографія Cholecystography

Фіброгастродуоденоскопія Fibrogastroduodenoscopy

40 / 200
Хворий 23-х років впродовж останніх 6-ти місяців став замкненим, уникав спілкування з оточуючими, усамітнювався, розмовляв сам з собою. При розмові з лікарем-психіатром спочатку приховував свої переживання, але потім розповів про особливі, нереальні голоси, що містяться в ньому самому ('гніздяться в мозку'). Які це розлади сприйняття? The 23-year-old patient became withdrawn over the past 6 months, avoided communication with others, became isolated, talked to himself. When talking to a psychiatrist, at first hid his experiences, but then told about special, unreal voices contained in himself ('nesting in the brain'). What are these perceptual disorders?

Псевдогалюцинації Pseudohallucinations

Дереалізація Derealization

Галюцинації справжні Hallucinations are real

Ілюзії Illusions

Метаморфопсії Metamorphopsia

41 / 200
У хлопчика 16-ти років без клінічної симптоматики при аускультації серця виявили акцент ІІ тону і систолічний шум на легеневій артерії. Тони серця звучні, ритмічні. Який найбільш імовірний діагноз? In a 16-year-old boy without clinical symptoms, an accent of the II sound and a systolic murmur on the pulmonary artery were detected during auscultation of the heart. Heart sounds are sonorous, rhythmic. What is the most likely diagnosis ?

Стеноз гирла легеневої артерії Stenosis of the mouth of the pulmonary artery

Незарощення боталової протоки Non-fouling of the botal channel

Функціональний шум Functional noise

Дефект міжпередсердної перегородки Atrial septal defect

Недостатність клапана легеневої артерії Pulmonary valve insufficiency

42 / 200
Дівчинка 10-ти років потрапила у відділення із проявами кардиту. В анамнезі: два тижні тому було загострення хронічного тонзиліту. Який етіологічний фактор кардиту найбільш імовірний у цьому випадку? A 10-year-old girl was admitted to the department with symptoms of carditis. In the anamnesis: there was an exacerbation of chronic tonsillitis two weeks ago. What is the most likely etiological factor of carditis in this case?'

Стафілокок Staphylococcus

Клебсієла Klebsiella

Протей Proteus

Пневмокок Pneumococcus

Стрептокок Streptococcus

43 / 200
У 3-річної дитини, що має дефіцит маси тіла, відзначається постійний вологий кашель. В анамнезі декілька перенесених пневмоній, які протікають з явищами обструкції. Об’єктивно: грудна клітка здута, над легенями вкорочення перкуторного звуку у нижніх відділах, аускультативно -велика кількість різнокаліберних вологих хрипів. Рівень хлоридів поту 80 ммоль/л. Вкажіть попередній діагноз: A 3-year-old child with a body weight deficit has a constant wet cough. He has several previous pneumonias in the anamnesis, which occur with obstruction phenomena. Objectively: the chest is inflated, over the lungs the percussion sound is shortened in the lower parts, auscultation - a large number of wet rales of various calibers. The level of sweat chlorides is 80 mmol/l. Specify the previous diagnosis:

Муковісцидоз Cystic Fibrosis

Бронхіальна астма Bronchial asthma

Рецидивуючий бронхіт Recurrent bronchitis

Бронхоектатична хвороба Bronchoectatic disease

Гіпоплазія легень Pulmonary hypoplasia

44 / 200
У хлопчика 1-го року після падіння на прогулянці з’явилася набряклість і болісність лівого гомілковоступневого суглоба. Раніше після невеликих травм відзначалися значні кровопідтікання. Тривалість кровотечі за Дюком - 3 хвилини. Згортання крові за ЛіУайтом - 24 хвилини. Яке захворювання можна припустити у хлопчика? A 1st-year-old boy developed swelling and pain in the left ankle joint after a fall on a walk. Previously, significant bleeding was noted after minor injuries. Duration of bleeding according to Duke - 3 minutes. Blood coagulation according to Lee White - 24 minutes. What disease can be assumed in the boy?

Гемофілія Hemophilia

ЮРА, переважно суглобова форма JURA, mainly articular form

Геморагічний васкуліт, суглобова форма Hemorrhagic vasculitis, articular form

Ідіопатична тромбоцитопенічна пурпура, гострий перебіг Idiopathic thrombocytopenic purpura, acute course

Ідіопатична тромбоцитопенічна пурпура, хронічний перебіг Idiopathic thrombocytopenic purpura, chronic course

45 / 200
На молочній фермі були відібрані проби молока на вміст хлоорганічного пестициду ліндану. Розмір партії 1000 літрів. У токсикологічній лабораторії СЕС було виявлено, що вміст ліндану перевищує максимально допустимі рівні у 4 рази. Дайте заключення про якість молока: On a dairy farm, milk samples were taken for the content of the organochlorine pesticide lindane. The batch size is 1000 liters. In the toxicological laboratory of SES, it was found that the content of lindane exceeds the maximum permissible levels in 4 times. Give a conclusion about the quality of milk:

Фальсифіковане Falsified

Умовно придатне Conditionally eligible

Недоброякісне Bad quality

Біологічно цінний продукт Biologically valuable product

Доброякісне Good quality

46 / 200
При проведенні проби на толерантність до глюкози дитині 13-ти років з ожирінням ІІІ ступеня встановлено: цукор крові натще - 5,4 ммоль/л, через 1 годину після вуглеводного навантаження - 10 ммоль/л, через 2 години - 7,8 ммоль/л. Які заходи необхідно провести щодо нормалізації вуглеводного обміну? During a glucose tolerance test for a 13-year-old child with third-degree obesity, the following was found: fasting blood sugar - 5.4 mmol/l, 1 hour after carbohydrate load - 10 mmol/l, after 2 hours - 7.8 mmol/l. What measures should be taken to normalize carbohydrate metabolism?

Препарати бігуанідного ряду Biguanide drugs

Дієта, руховий режим Diet, movement mode

Сульфаніламідні препарати Sulphanilamide preparations

Інсулін Insulin

Цукрознижуючі збори трав Sagar-lowering herbal collections

47 / 200
У 10-річного хлопчика на фоні вживання еритроміцину гостро розвинулися ознаки медикаментозної алергії у вигляді генералізованої кропив’янки та масивного набряку Квінке. Стан дитини важкий. З яких препаратів слід розпочати інтенсивну терапію? A 10-year-old boy, on the background of erythromycin use, acutely developed signs of drug allergy in the form of generalized urticaria and massive Quincke's edema. The child's condition is serious. Which drugs should start intensive therapy?

Седативні Sedatives

Діуретики Diuretics

Антигістамінні Antihistamines

Глюкокортикоїди Glucocorticoids

Кальцію глюконат Calcium gluconate

48 / 200
Першородяча 30-ти років. Голівка у порожнині малого тазу. Серцебиття плоду почало уповільнюватися, з’явилася аритмія. Що робити? 30-year-old first-born. The head is in the pelvic cavity. The fetal heartbeat began to slow down, arrhythmia appeared. What to do?

Перінеотомія Perineotomy

Шкірно-головні щипці Scalp forceps

Кесарів розтин Caesarean section

Вихідні акушерські щипці Outgoing obstetric forceps

Порожнинні акушерські щипці Hollow obstetric forceps

49 / 200
В районі розповсюдженість захворювань серед населення становила 1156 випадків на 1000 населення. Який з приведених нижче показників характеризує розповсюдженість захворювань? In the region, the prevalence of diseases among the population was 1156 cases per 1000 population. Which of the following indicators characterizes the prevalence of diseases?

Стандартизований Standardized

Екстенсивний Extensive

Інтенсивний Intensive

Наочності Visibility

Співвідношення Ratio

50 / 200
У новонародженого з підозрою на внутрішньочерепну пологову травму проведено люмбальну пункцію. Було отримано кров’янистий ліквор. Який крововилив має місце у цьому випадку? A newborn with suspected intracranial birth trauma underwent a lumbar puncture. A bloody cerebrospinal fluid was obtained. What type of hemorrhage occurs in this case?

Субтенторіальний Subtentorial

Супратенторіальний Supratentorial

Субарахноїдальний Subarachnoid

Кефалогематома Cephalohematoma

Епідуральний Epidural

51 / 200
Хвора 47-ми років страждає на гіпомоторну дискінезію товстого кишечнику. Як модернізувати харчовий раціон з метою підвищення моторики кишечнику? A 47-year-old patient suffers from hypomotor dyskinesia of the large intestine. How to modernize the diet in order to increase intestinal motility?

У гіпоенергетичній дієті збільшити кількість рослинних білків, молочних продуктів, кальцію In a hypoenergetic diet, increase the amount of vegetable proteins, dairy products, calcium

У раціональній дієті збільшити кількість рослинного волокна та кисломолочних продуктів In a rational diet, increase the amount of vegetable fiber and fermented milk products

У раціональній дієті збільшити кількість рослинних, кисломолочних продуктів та магнію In a rational diet, increase the number of vegetable, sour-milk products and magnesium

У раціональній дієті збільшити кількість рослинних білків, тваринних жирів, калію In a rational diet, increase the amount of vegetable proteins, animal fats, potassium

У редукуючій білково-овочевій дієті збільшити кількість тваринних жирів та магнію In a reducing protein-vegetable diet, increase the amount of animal fats and magnesium

52 / 200
Хлопчик 4-х років госпіталізований до стаціонару зі скаргами на задишку, швидку втомлюваність. В анамнезі часті респіраторні захворювання. Перкуторно: межі серця розширені вліво та вгору. Аускультатив-но: посилення ІІ тону над легеневою артерією, в ІІ-ІІІ міжребер’ї ліворуч від груднини вислуховується грубий систолодіастолічний 'машинний'шум, що проводиться в усі інші точки та на спину. Який найбільш вірогідний діагноз? A 4-year-old boy was hospitalized with complaints of shortness of breath, rapid fatigue. He has a history of frequent respiratory diseases. Percussion: the borders of the heart are expanded to the left and up. Auscultative- but: strengthening of the II tone over the pulmonary artery, in the II-III intercostal space to the left of the sternum, a rough systolodiastolic 'machine' noise is heard, which is carried to all other points and to the back. What is the most likely diagnosis?

Ізольований стеноз отвору легеневої артерії Isolated stenosis of the opening of the pulmonary artery

Клапанний стеноз аорти Valvular stenosis of the aorta

Відкрита артеріальна протока Open ductus arteriosus

Дефект міжпередсердної перегородки Atrial septal defect

Дефект міжшлуночкової перегородки Ventricular septal defect

53 / 200
Жінка 35-ти років скаржиться на біль у ділянці серця ('щемить, свердлить'), що виникає переважно у ранкові години в осінньо-весняний період, з іррадіацією болю в шию, спину, живіт; часте серцебиття, а також зниження загального життєвого тонусу. Виникнення цього стану не зв’язане з фізичним навантаженням. Увечері стан поліпшується. Соматичний, неврологічний статус та ЕКГ - без патології. Яка найбільш імовірна патологія зумовила таку клінічну картину? A 35-year-old woman complains of pain in the area of the heart ('pinching, drilling'), which occurs mainly in the morning hours in the autumn-spring period, with irradiation pain in the neck, back, abdomen; frequent heartbeat, as well as a decrease in general vitality. The occurrence of this condition is not associated with physical exertion. In the evening, the condition improves. Somatic, neurological status and ECG - without pathology. What is the most likely pathology that caused such a clinical picture?

Нейроциркуляторна дистонія Neurocirculatory dystonia

Неврозоподібна шизофренія Neurosis-like schizophrenia

Соматизована депресія Somatized depression

Іпохондрична депресія Hypochondriac depression

Стенокардія спокою Angina at rest

54 / 200
Хворий 42-х років скаржиться на серцебиття, постійну слабкість, болі в ділянці серця, що іррадіюють у ліве плече та підсилюються при диханні, підвищення температури тіла до 38, 5oC Об’єктивно: АТ-105/50 мм рт.ст., Ps- 120/хв. У зоні абсолютної тупості серця вислуховується двофазний шум. В легенях дихання послаблене в нижніх відділах. На ЕКГ - конкордантний підйом сегменту ST у V2 -V6 . Який найбільш імовірний діагноз? A 42-year-old patient complains of palpitations, constant weakness, pain in the heart region radiating to the left shoulder and getting worse when breathing, an increase in body temperature to 38, 5oC Objectively: BP-105/50 mm Hg, Ps- 120/min. In the zone of absolute dullness of the heart, a biphasic murmur is heard. In the lungs, breathing is weakened in the lower parts. On the ECG, there is a concordant elevation of the ST segment in V2-V6 . What is the most likely diagnosis?

Позагоспітальна пневмонія Out-of-hospital pneumonia

Інфаркт легень Pulmonary infarction

Гострий перикардит Acute pericarditis

Синдром Дреслера Dresler syndrome

Інфаркт міокарда Myocardial infarction

55 / 200
Хворий 29-ти років захворів гостро 3 тижні тому. Різко підвищилась температура, з’явились сильні болі в лівій гомілці. На рентгенограмах цієї ділянки множинні вогнища деструкції неправильної геометричної форми з нерівними контурами. В ділянках деструкції визначаються щільні тіні секвестрів. Навколо уражених відділів кістки визначаються смужки звапнення відшарованого окістя. Який найбільш імовірний діагноз? A 29-year-old patient became acutely ill 3 weeks ago. The temperature rose sharply, severe pain appeared in the left lower leg. On the radiographs of this area, multiple foci of destruction of an irregular geometric shapes with uneven contours. Dense shadows of sequestrations are defined in areas of destruction. Strips of calcification of exfoliated periosteum are defined around the affected parts of the bone. What is the most likely diagnosis?

Туберкульозне ураження кісток лівої гомілки Tuberculous lesion of the bones of the left leg

Саркома лівої великогомілкової кістки Sarcoma of the left tibial bone

Фіброзна остеодистрофія кісток лівої гомілки Fibrous osteodystrophy of the bones of the left tibia

Гострий гематогенний остеомієліт кісток лівої гомілки Acute hematogenous osteomyelitis of the bones of the left tibia

Метастатичне ураження кісток лівої гомілки Metastatic lesions of the bones of the left tibia

56 / 200
Хворий 20-ти років скаржиться на сильний головний біль, двоїння в очах, загальну слабкість, підвищення температури тіла, дратівливість. Об’єктивно: температура тіла 38,1°С, в контакт вступає неохоче, болісно реагує на подразники. Птоз лівої повіки, розбіжна косоокість, анізокорія S>D Виражений менінгеальний синдром. При люмбальній пункції ліквор витікав під тиском 300 мм вод.ст., прозорий, з легкою опалесценцією, через добу випала фібринозна плівка. Білок -1,4 г/л, лімфоцити - 600/3 в мм3 , цукор - 0,3 ммоль/л. Який попередній діагноз? A 20-year-old patient complains of severe headache, double vision, general weakness, increased body temperature, irritability. Objectively: body temperature 38.1 °С, comes into contact reluctantly, reacts painfully to stimuli. Ptosis of the left eyelid, divergent strabismus, anisocoria S>D Pronounced meningeal syndrome. During a lumbar puncture, cerebrospinal fluid leaked out under a pressure of 300 mm Hg, transparent, with slight opalescence, after a day a fibrinous film fell out. Protein - 1.4 g/l, lymphocytes - 600/3 in mm3, sugar - 0.3 mmol/l. What is the previous diagnosis?

Паротитний менінгіт Mumps meningitis

Лімфоцитарний менінгіт Армстронга Armstrong lymphocytic meningitis

Туберкульозний менінгіт Tuberculous meningitis

Сифілітичний менінгіт Syphilitic meningitis

Менінгококовий менінгіт Meningococcal meningitis

57 / 200
Голівка новонародженого має доліхоцефалічну форму, витягнута спереду назад. Під час огляду голівки на потиличній частині визначається пологова пухлина, розташована на середині між великим і малим тім’ячком. При якому передлежанні голівки плоду відбулися пологи? The head of a newborn has a dolichocephalic shape, elongated from front to back. During the examination of the head on the occipital part, a birth tumor is determined, located in the middle between the large and small parietal. to which presentation of the fetal head did the birth take place?

Передньо-тім’яне передлежання Anterior-parietal presentation

Задній вид потиличного передлежання Posterior view of occipital presentation

Передній вид потиличного передлежання Anterior view of occipital presentation

Лобне передлежання Frontal presentation

Лицеве передлежання Face-to-face

58 / 200
У хворого з явищами порушеного харчування, стеатореї, через 4 години після їжі виникає біль у животі, особливо вище пупка і лівіше. Проноси можуть змінитися закрепами до 3-5 днів. Пальпаторно: помірна болісність у холедохопанкреатичній зоні. Рівень амілази у крові не підвищується. При УЗД виявлені кальцинати, що розташовані в ділянці підшлункової залози. Який попередній діагноз? A patient with symptoms of impaired nutrition, steatorrhea, 4 hours after eating, abdominal pain occurs, especially above the navel and to the left. Diarrhea can change to constipation up to 3-5 days. Palpation: moderate pain in the choledochopancreatic area. The level of amylase in the blood does not increase. Ultrasound revealed calcifications located in the area of the pancreas. What is the previous diagnosis?

Хронічний панкреатит Chronic pancreatitis

Синдром Цолінгер-Елісона Zolinger-Ellison syndrome

Хронічний калькульозний холецистит Chronic calculous cholecystitis

Виразка 12-палої кишки Duodenal ulcer

Хронічний гастродуоденіт Chronic gastroduodenitis

59 / 200
Хворий 54-х років скаржиться на часте болісне сечовипускання, озноби, підвищення температури тіла до 38°С. У сечі: білок -0,33 г/л, лейкоцити до 50-60 у п/з, еритроцити - 5-8 у п/з, грамнегативні палички. Який з представлених антибактеріальних препаратів найкращий у цьому випадку? A 54-year-old patient complains of frequent painful urination, chills, an increase in body temperature up to 38°C. In the urine: protein -0.33 g/l, leukocytes up to 50-60 in p/z, erythrocytes - 5-8 in p/z, gram-negative rods. Which of the presented antibacterial drugs is the best in this case?

Тетрациклін Tetracycline

Оксацилін Oxacillin

Цепорін Tseporin

Еритроміцин Erythromycin

Ципрофлоксацин Ciprofloxacin

60 / 200
У пологовому будинку у дитини на 3-й день життя з’явилася геморагічна висипка, блювання з кров’ю, випорожнення чорного кольору. Обстеження виявило анемію, подовження часу згортання крові, гіпопротромбінемію, нормальну кількість тромбоцитів. Яка оптимальна терапевтична тактика? In the maternity hospital, on the 3rd day of life, the child developed a hemorrhagic rash, vomiting blood, black stools. Examination revealed anemia, prolonged blood coagulation, hypoprothrombinemia, normal platelet count. What is the optimal therapeutic strategy?

Вітамін К Vitamin K

Фібриноген Fibrinogen

Гііюконат кальцію Calcium hydroxyconate

Епсілон-амінокапронова кислота Epsilon-aminocaproic acid

Етамзилат натрію Etamsylate sodium

61 / 200
На диспансерному обліку в сімейного лікаря знаходяться реконвалесценти після інфекційних захворювань, часто та тривало хворіючи, особи з хронічною патологією. Хто, з наведених хворих, має бути віднесений до III групи здоров’я? There are convalescents after infectious diseases, often and long-term illness, persons with chronic pathology on the dispensary register of the family doctor. Which of the listed patients should be classified as III health groups?

Хронічна патологія та ті, що часто і тривало хворіють Chronic pathology and those who are sick often and for a long time

Особи, що страждають на хронічні захворювання Persons suffering from chronic diseases

Усі категорії хворих, що наведені в умовах All categories of patients specified in the conditions

Реконвалесценти після інфекційних захворювань та особи з хронічною патологією Convalescents after infectious diseases and persons with chronic pathology

Ті, що часто та тривало хворіють Those who get sick often and for a long time

62 / 200
Хворий 62-х років звернувся зі скаргами на періодичне відходження червоподібних кров’яних згустків із сечею. У правій половині живота під час пальпації визначається горбисте, безболісне, рухливе утворення. Який з перерахованих методів обстеження необхідно застосувати у першу чергу? A 62-year-old patient complained of periodic discharge of worm-shaped blood clots in the urine. In the right half of the abdomen, during palpation, a lumpy, painless, mobile mass is determined Which of the listed examination methods should be used first?

Екскреторну урографію Excretory urography

УЗД нирок та заочеревинного простору Ultrasound of kidneys and retroperitoneal space

Цистоскопію Cystoscopy

Хромоцистоскопію Chromocystoscopy

Комп’ютерну томографію тазу Computed tomography of the pelvis

63 / 200
Жінка 42-х років скаржиться на появу синців на ногах та тривалі менструації, загальну слабкість, шум у голові. Об’єктивно: велика кількість плямистих геморагій на ногах і тулубі. Тахіпное, тахікардія, систолічний шум у всіх точках. АТ- 75/50 мм рт.ст. У крові: ер.-1,9-1012/л, НЬ- 60 г/л, КП-0,9, лейк.- 6, 5 -109 /л, тр.- 20 -109 /л, ШОЕ-12 мм/год. Тривалість кровотечі за Дуке - 12 хвилин. У кістковому мозку - велика кількість молодих незрілих форм мегакаріобластів без ознак відшнурування тромбоцитів. Який найбільш імовірний діагноз? A 42-year-old woman complains of bruises on her legs and prolonged menstruation, general weakness, noise in her head. Objectively: a large number of spotty hemorrhages on her legs and trunk. Tachypnea, tachycardia, systolic murmur at all points. Blood pressure - 75/50 mm Hg. In blood: ER - 1.9-1012/l, Hb - 60 g/l, CP - 0.9, leuk .- 6, 5 -109 /l, tr.- 20 -109 /l, ESR-12 mm/h. Duration of bleeding according to Duque - 12 minutes. In the bone marrow - a large number of young, immature forms of megakaryoblasts without signs of platelet detachment. What the most likely diagnosis?

Гемофілія В Hemophilia B

Хвороба Віллебрандта Willebrandt's disease

Гострий мегакаріобластний лейкоз Acute megakaryoblastic leukemia

Гемофілія А Hemophilia A

Ідеопатична тромбоцитопенічна пурпура Ideopathic thrombocytopenic purpura

64 / 200
Головний лікар поліклініки дав завдання лікарю загальної практики - сімейної медицини визначити захворюваність N-ою хворобою серед дорослого населення. Який документ дасть можливість визначити патологічну ураженість населення? The head doctor of the polyclinic gave the task to the doctor of general practice - family medicine to determine the incidence of the Nth disease among the adult population. What document will make it possible to determine the pathological lesion of the population?

Медична карта амбулаторного пацієнта Medical card of an outpatient

Талон для реєстрації заключних (уточнених) діагнозів зі знаком (-) Ticket for registration of final (specified) diagnoses with a (-) sign

Талон для реєстрації заключних (уточнених) діагнозів зі знаком (+) і (-) Ticket for registration of final (specified) diagnoses with a (+) and (-) sign

Талон для реєстрації заключних (уточнених) діагнозів зі знаком (+) Ticket for registration of final (specified) diagnoses with (+) sign

Журнал профілактично оглянутих Log of preventive inspections

65 / 200
Чоловік 68-ми років скаржиться на кашель із виділенням харкотиння, який турбує його впродовж декількох років, осиплість голосу, загальну слабкість. Проживає поблизу заводу по переробці азбесту. Об’єктивно: в легенях справа - ослаблене дихання з подовженим видихом, сухі хрипи. На рентгенограмі: в ділянці кореня і прикореневої зони справа неоднорідне, з нечіткими контурами затемнення, підвищена повітряність легень. ЛОР - парез правої голосової зв’язки. Який найбільш імовірний діагноз? A 68-year-old man complains of a cough with expectoration that has bothered him for several years, hoarseness of voice, general weakness. He lives near an asbestos processing plant. About objectively: in the lungs on the right - weakened breathing with prolonged exhalation, dry wheezes. On the X-ray: in the area of the root and basal zone on the right, there is a heterogeneous darkening with unclear contours, increased airiness of the lungs. ENT - paresis of the right vocal cord. What is the most likely diagnosis ?

Хронічний пиловий бронхіт Chronic dust bronchitis

Азбестоз Asbestosis

Туберкульоз легень Pulmonary tuberculosis

Центральний рак правої легені Central right lung cancer

Правобічна прикоренева пневмонія Right-sided basal pneumonia

66 / 200
Жінка 58-ми років скаржиться на безпричинну появу синців, слабкість, кровоточивість ясен, запаморочення. Об’єктивно: слизові оболонки та шкірні покриви бліді, з численними крововиливами різної давнини. Лімфатичні вузли не збільшені. Рs- 100/хв., АТ-110/70 мм рт.ст. З боку внутрішніх органів змін не виявлено. У крові: ер.- 3,0-1012/л, НЬ- 92 г/л, КП- 0,9, анізоцитоз, пойкілоци-тоз, лейк.- 10-109 /л, е- 2%, п- 12%, с- 68%, л11%, м- 7%, ШЗЕ- 12 мм/год. Додаткове визначення якого лабораторного показника найбільш доцільне для встановлення діагнозу? A 58-year-old woman complains of unexplained bruising, weakness, bleeding gums, dizziness. Objectively: mucous membranes and skin are pale, with numerous hemorrhages of various of ancient times. Lymph nodes are not enlarged. Ps - 100/min., BP - 110/70 mm Hg. No changes were detected on the part of internal organs. In blood: ER - 3.0-1012/l, Hb - 92 g /l, CP- 0.9, anisocytosis, poikilocytosis, leuk.- 10-109 /l, e- 2%, p- 12%, c- 68%, l11%, m- 7%, SZE- 12 mm/h. Additional determination of which laboratory indicator is the most appropriate for establishing a diagnosis?

Тромбоцити Platelets

Ретикулоцити Reticulocytes

Фібриноген Fibrinogen

Час згортання крові Blood coagulation time

Осмотична резистентність еритроцитів Osmotic resistance of erythrocytes

67 / 200
Жінка 34-х років скаржиться на слабкість, зниження ваги на 12 кг за півроку, пітливість, серцебиття, дратівливість. Об’єктивно: щитоподібна залоза III ступеню, еластична, на фоні дифузного збільшення в правій частці вузол. Шийні лімфатичні вузли не збільшені. Яка лікувальна тактика найбільш обґрунтована? A 34-year-old woman complains of weakness, weight loss of 12 kg in six months, sweating, palpitations, irritability. Objectively: thyroid gland III degree, elastic , on the background of a diffuse increase in the right lobe of the nodes. The cervical lymph nodes are not enlarged. What treatment tactics are the most justified?

Операція після антитиреоїдної терапії Surgery after antithyroid therapy

Негайна телегаматерапія Immediate telegamotherapy

Негайне хірургічне втручання Immediate surgical intervention

Призначення радіоактивного йоду Purpose of radioactive iodine

Консервативна антитиреоїдна терапія Conservative antithyroid therapy

68 / 200
Чоловік 47-ми років страждає на рак легень в термінальній стадії. Розповідає, що протягом останніх 2-3-х тижнів помітив зміни в своєму психічному стані: бачить ввече- рі покійну матір, яка кличе до себе, відчуває запах сирої землі, гнилого листя, бачить незнайомих людей, які несуть труну. При цьому відчуває страх. Які симптоми психічних розладів у хворого? A 47-year-old man suffers from lung cancer in the terminal stage. He says that during the last 2-3 weeks he has noticed changes in his mental state: he sees in the evening 'I see my late mother calling to me, smelling raw earth, rotten leaves, seeing strangers carrying a coffin. At the same time, I feel fear. What are the symptoms of mental disorders in the patient?

Депресивні розлади Depressive disorders

Психосенсорні розлади Psychosensory disorders

Ілюзорні розлади Illusory Disorders

Маячні ідеї Delusive ideas

Справжні галюцинації True hallucinations

69 / 200
Хворий 60-ти років надійшов до клініки зі скаргами на задишку, важкість у правому підребер’ї, збільшення живота. Явища зростали протягом року. При аускультації серця - ритм галопу. Об’єктивно: набухання шийних вен, асцит, пальпується печінка та селезінка. З яким захворюванням необхідно проводити диференційну діагностику? A 60-year-old patient came to the clinic with complaints of shortness of breath, heaviness in the right hypochondrium, abdominal enlargement. The phenomena increased during the year. During auscultation of the heart - rhythm gallop. Objectively: swelling of the jugular veins, ascites, palpable liver and spleen. What disease should be differentially diagnosed?

Цироз печінки Liver cirrhosis

Констриктивний перикардит Constrictive pericarditis

Тромбоемболія легеневої артерії Thromboembolism of the pulmonary artery

Хронічне легеневе серце Chronic pulmonary heart

Рак легенів з проростанням у плевру Lung cancer with proliferation in the pleura

70 / 200
До травмпункту звернувся фермер, який під час роботи в полі поранив праву ступню. Коли був щепленим, не пам’ятає, в армії не служив. При огляді правої ступні виявлено забруднену рану довжиною 5-6 см з нерівним краями. Яка подальша тактика лікаря? A farmer who injured his right foot while working in the field came to the emergency department. He does not remember when he was vaccinated, he did not serve in the army. When examining his right foot a contaminated wound 5-6 cm long with uneven edges was detected. What is the doctor's further tactics?

Ввести протиправцеву сироватку Enter anti-tetanus serum

Тільки хірургічна обробка рани Only surgical wound treatment

Ввести правцевий анатоксин Enter tetanus toxoid

Призначити антибіотик Prescribe antibiotic

Ввести правцевий анатоксин і протиправцеву сироватку Enter tetanus toxoid and anti-tetanus serum

71 / 200
У хворого 50-ти років раптово виник біль у потиличній ділянці, блювання. Об’єктивно: сопор, гіперемія шкіри обличчя, АТ- 210/120 мм рт.ст., Рs- 60/хв, напружений, температура тіла - 37,8°Є. Горизонтальний ністагм. Виражені рефлекси орального автоматизму. Сухожилкові рефлекси рівномірні. Ригідність потиличних м’язів, двобічний симптом Керніга. Який попередній діагноз? A 50-year-old patient suddenly developed pain in the back of the head, vomiting. Objectively: sopor, hyperemia of the skin of the face, blood pressure - 210/120 mm Hg. st., Рs- 60/min, tense, body temperature - 37.8°С. Horizontal nystagmus. Pronounced reflexes of oral automatism. Tendon reflexes are even. Rigidity of occipital muscles, bilateral Kernig symptom. What is the previous diagnosis?

Менінгококовий менінгіт Meningococcal meningitis

Геморагічний паренхіматозний інсульт Hemorrhagic parenchymal stroke

Гостра гіпертонічна енцефалопатія Acute hypertensive encephalopathy

Субдуральна гематома Subdural hematoma

Субарахноїдальний крововилив Subarachnoid hemorrhage

72 / 200
У доношеної дитини віком 6 днів на різних ділянках шкіри виявляються еритема, мляві пухирі, ерозивні поверхні, тріщини, лущення епідермісу, які виглядають ніби після ошпарення окропом. Виявлено позитивний симптом Нікольського. Загальний стан дитини важкий. Виражений неспокій, гіперестезія, фебрильна температура. Який найбільш імовірний діагноз у цьому випадку? A 6-day-old full-term child has erythema, flaccid blisters, erosive surfaces, cracks, flaking of the epidermis, which look as if after being scalded with boiling water, on various areas of the skin. Positive Nikolsky's symptom. The general condition of the child is severe. Pronounced restlessness, hyperesthesia, febrile temperature. What is the most likely diagnosis in this case?

Псевдофурункульоз Фігнера Pseudofurunculosis Figner

Флегмона новонародженого Phlegmon of a newborn

Ексфоліативний дерматит Ріттера Ritter's exfoliative dermatitis

Епідермоліз Epidermolysis

Пухирчатка новонародженого Neonatal pemphigus

73 / 200
Дівчинка 8-ми років госпіталізована до кардіологічного відділення. Об’єктивно: відзначається ураження шкіри над розгинальними поверхнями суглобів з атрофічними рубчиками, депігментація, симетричне ураження скелетних м’язів (слабкість, біль, набряки, гіпотрофія). Для якого захворювання найбільш характерні вказані зміни? An 8-year-old girl was hospitalized in the cardiology department. Objectively: skin damage over the extensor surfaces of the joints with atrophic scars, depigmentation, symmetrical skeletal muscle damage is noted (weakness, pain, swelling, hypotrophy). For which disease are the indicated changes most characteristic?

Дерматоміозит Dermatomyositis

Вузликовий періартеріїт Nodular periarteritis

Хвороба Рейтера Reuter's disease

Системна склеродермія Systemic scleroderma

Системний червоний вівчак System red sheep

74 / 200
Дитина 8-ми років, що страждає протягом 3-х років на цукровий діабет, доставлена у відділення в стані гіперглікемічної коми. Первинну дозу інсуліну слід призначити із розрахунку: An 8-year-old child who has been suffering from diabetes for 3 years was brought to the department in a hyperglycemic coma. The initial dose of insulin should be prescribed based on:

0,05 ОД/кг маси тіла на годину 0.05 units/kg of body weight per hour

0,1-0,2 ОД/кг маси тіла на годину 0.1-0.2 units/kg of body weight per hour

0,3-0,4 ОД/кг маси тіла на годину 0.3-0.4 units/kg of body weight per hour

0,4-0,5 ОД/кг маси тіла на годину 0.4-0.5 units/kg of body weight per hour

0,2-0,3 ОД/кг маси тіла на годину 0.2-0.3 units/kg of body weight per hour

75 / 200
До хірургічного відділення через 4 години після травми надійшов потерпілий 37-ми років з великою розчавленою раною лівого стегна. Яка головна умова успішної профілактики газової гангрени у потерпілого? 4 hours after the injury, a 37-year-old victim came to the surgical department with a large crushed wound of the left thigh. What is the main condition for the successful prevention of gas gangrene in the victim?

Введення специфічної сироватки 30 000 ОД Introduction of specific serum 30,000 units

Введення специфічної сироватки 3 000 ОД Introduction of specific serum 3,000 units

Промивання рани розчином перекису водню 6% Wound washing with 6% hydrogen peroxide solution

Видалення нежиттєздатних тканин та своєчасна хірургічна обробка рани Removal of nonviable tissues and timely surgical treatment of the wound

Інфільтрація м’яких тканин навкруги рани розчином антибіотиків Infiltration of soft tissues around the wound with a solution of antibiotics

76 / 200
Хворий 24-х років скаржиться на загальну слабкість, запаморочення, підвищення температури до 37, 5°С, біль у горлі, набряк шиї, збільшення підщелепних лімфовузлів. Об’єктивно: слизова ротоглотки набрякла та ціанотична, мигдалики збільшені, вкриті плівками, що поширюються за їх межі, знімаються важко. Який основний механізм розвитку даного захворювання? A 24-year-old patient complains of general weakness, dizziness, temperature rise to 37.5°C, sore throat, neck swelling, and enlarged submandibular lymph nodes. Objectively: the mucous membrane of the oropharynx is swollen and cyanotic, the tonsils are enlarged, covered with films that spread beyond their borders, and are difficult to remove. What is the main mechanism of the development of this disease?

Алергічний компонент Allergic component

Порушення метаболізму Metabolism disorders

Дія бактеріального екзотоксину Effect of bacterial exotoxin

Дія бактеріального ендотоксину Effect of bacterial endotoxin

Дизбіотичні зміни Dysbiotic changes

77 / 200
Пацієнт 38-ми років висловлює скарги на сильний пекучий біль у ділянці кисті та передпліччя. В анамнезі: різана рана волярної поверхні нижньої третини передпліччя. Пошкодження якого нерва найчастіше викликає подібні болі? A 38-year-old patient complains of severe burning pain in the area of the hand and forearm. History: cut wound of the volar surface of the lower third of the forearm. Damage to which nerve most often causes similar pains?

Міжкістковий Interosseous

Променевий Radiant

Серединний Middle

Ліктьовий Cubit

Пахвовий Axillary

78 / 200
Хворий після аварії скаржиться на біль у кульшовому суглобі. Нога в положенні згинання, приведення та внутрішньої ротації, значно скорочена. Пружний опір при спробі пасивного приведення чи відведення кінцівки. Великий вертлюг розташований високо над розернелатонівською лінією. Виявляється значний лордоз. Який попередній діагноз? After an accident, the patient complains of pain in the hip joint. The leg is in a position of flexion, adduction and internal rotation, significantly shortened. Elastic resistance when trying to passively adduct or abduct the limb. The greater rotator cuff is located high above the Rosernelaton line. There is significant lordosis. What is the preliminary diagnosis?

Перелом кульшової западини з центральним вивихом стегна Fracture of the acetabulum with central hip dislocation

Черезвертлюговий перелом стегна Transvertebral hip fracture

Перелом шийки стегнової кістки із зміщенням Displaced femoral neck fracture

Клубовий вивих стегна Club hip dislocation

Сідничний вивих стегна Gluteal hip dislocation

79 / 200
Через 3 тижні після перенесеної гострої ангіни у хворого залишаються слабкість, кволість, субфебрильна температура, збільшені позащелепні лімфатичні вузли. Мигдалики пухкі, спаяні з дужками, в лакунах - гнійні пробки. Який найбільш вірогідний діагноз? 3 weeks after the acute angina, the patient remains weak, frailty, low-grade fever, enlarged extramaxillary lymph nodes. Tonsils are loose, soldered with brackets, purulent in the lacunae traffic jams. What is the most likely diagnosis?

Паратонзиліт Paratonsillitis

Хронічній фарингіт Chronic pharyngitis

Хронічний тонзиліт Chronic tonsillitis

Пухлина мигдаликів Tonsil tumor

Гостра лакунарна ангіна Acute lacunar angina

80 / 200
Хворий 28-ми років скаржиться на біль у животі ріжучого характеру. Захворів різко 1,5 години тому, коли відчув гострий біль під грудьми. До цього нічим не хворів. Об’єктивно: шкіра бліда, хворий лежить на спині. При зміні положення тіла біль загострюється. Р8- 70/хв, ритмічний, АТ- 100/60 мм рт.ст. З боку серця та легень патології немає. Язик сухий, чистий. Живіт втягнутий, не бере участі в диханні, під час пальпації різко напружений та болісний у всіх відділах. Симптом Щоткіна-Блюмберга позитивний. Який найбільш імовірний діагноз? A 28-year-old patient complains of a sharp abdominal pain. He became ill 1.5 hours ago, when he felt a sharp pain under the chest. Before that, he was not sick with anything . Objectively: the skin is pale, the patient lies on his back. When the body position changes, the pain worsens. P8 - 70/min, rhythmic, BP - 100/60 mm Hg. There is no pathology on the part of the heart and lungs. The tongue is dry, clean . The abdomen is drawn in, does not participate in breathing, during palpation it is sharply tense and painful in all departments. The Shtokkin-Blumberg symptom is positive. What is the most likely diagnosis?

Гострий апендицит Acute appendicitis

Гострий холецистит Acute cholecystitis

Перфоративна виразка шлунка Perforative gastric ulcer

Гострий панкреатит Acute pancreatitis

Гостра кишкова непрохідність Acute intestinal obstruction

81 / 200
Дитина знаходиться на стаціонарному лікуванні з приводу гострої стафілококової деструкції правої легені. Раптово з’явилися гострий біль у правій половині грудної клітки, задишка, ціаноз. Права половина грудної клітки відстає в акті дихання. Перкуторно справа внизу - тупість, в верхніх відділах - коробковий звук. Межі відносної серцевої тупості зміщені вліво. Яке ускладнення найбільш імовірно виникло у дитини? The child is under inpatient treatment for acute staphylococcal destruction of the right lung. Acute pain in the right half of the chest suddenly appeared, shortness of breath, cyanosis. The right half of the chest lags behind in the act of breathing. Percussion in the lower right - dullness, in the upper parts - a box sound. The limits of relative cardiac dullness are shifted to the left. What complication most likely occurred in the child?

Абсцес правої легені Abscess of the right lung

Спонтанний пневмоторакс Spontaneous pneumothorax

Піопневмоторакс справа Pyopneumothorax on the right

Емпієма плеври Empyema of the pleura

Ексудативний плеврит Exudative pleurisy

82 / 200
На амбулаторний прийом до сімейного лікаря звернувся пацієнт, якому після обстеження було встановлено діагноз ГРЗ. Який обліковий документ слід заповнити лікарю, щоб зареєструвати цей випадок захворювання? A patient who, after examination, was diagnosed with acute coronary syndrome went to the family doctor for an outpatient appointment. What accounting document should the doctor fill out in order to register this case of the disease?

Талон на прийом до лікаря Coupon for an appointment with a doctor

Екстрене повідомлення про інфекційне захворювання Emergency message about an infectious disease

Статистичний талон для реєстрації заключного діагнозу Statistical coupon for registration of final diagnosis

Листок непрацездатності Sick leave

Медична карта амбулаторного хворого Medical card of an outpatient

83 / 200
Дівчинка 12-ти років відмічає кровотечу із статевих шляхів останні 2 тижні, після затримки менструації впродовж 3-х місяців, слабкість, головний біль, запаморочення. Менструації з 10 років. Об’єктивно: шкірні покриви бліді, тахікардія, АТ- 100/60 мм рт.ст. У крові: Нb-100 г/л, тромбоцити 200 • 109 /л. Гінекологічне обстеження: virgo. При ректальному обстеженні: тіло матки і придатки без патології. Який найбільш імовірний діагноз? A 12-year-old girl notices bleeding from the genital tract for the last 2 weeks, after a delay of menstruation for 3 months, weakness, headache, dizziness. Menstruation from 10 years old. Objectively: the skin is pale, tachycardia, blood pressure - 100/60 mm Hg. In the blood: Hb - 100 g/l, platelets 200 • 109 /l. Gynecological examination: virgo. During rectal examination: the body of the uterus and appendages without pathology. What is the most likely diagnosis?

Хвороба Верльгофа Werlhof's disease

Синдром склерокістозних яєчників Syndrome of sclerocystic ovaries

Ювенільна кровотеча Juvenile bleeding

Перервана вагітність Interrupted pregnancy

Геморагічний діатез Hemorrhagic diathesis

84 / 200
Дитині 1 доба. У пологах утруднене виведення плечиків. Маса тіла 4300 г. Права рука звисає вздовж тулуба, кисть пронована, рухи у руці відсутні. Позитивний симптом шарфа. Вкажіть найбільш імовірний діагноз: The child is 1 day old. During childbirth, it is difficult to withdraw the shoulders. Body weight is 4300 g. The right arm hangs along the body, the hand is pronated, there are no movements in the arm. Positive symptom of scarf. Specify the most likely diagnosis:

Тетрапарез Tetraparesis

Тотальний акушерський параліч справа Total obstetric paralysis on the right

Геміпарез Hemiparesis

Проксимальний тип акушерського паралічу справа Proximal type of obstetric paralysis on the right

Дистальний тип акушерського паралічу справа Distal type of obstetric paralysis on the right

85 / 200
Працівникові, який знаходиться на диспансерному спостереженні з приводу виразкової хвороби 12-палої кишки, на підприємстві виділили пільгову путівку для санаторнокурортного лікування на 24 дні. Тривалість щорічної основної відпустки працівника складає 24 календарних дні, на проїзд до санаторію і назад необхідно витратити 4 дні. Який порядок оформлення тимчасової непрацездатності в цьому випадку? The employee, who is under observation at the dispensary for duodenal ulcer, was allocated a discounted ticket for sanatorium treatment for 24 days at the enterprise. The duration of the employee's annual basic leave is 24 calendar days, it is necessary to spend 4 days traveling to the sanatorium and back. What is the procedure for registering temporary disability in this case?

Лікуючий лікар видає довідку і 'Санаторно-курортну карту' на 28 днів The attending physician issues a certificate and a 'Sanatorium-resort card' for 28 days

Лікуючий лікар видає листок непрацездатності на 4 дні The attending physician issues a sick leave for 4 days

Через ЛКК видається довідка на 4 дні A certificate for 4 days is issued through LKK

Через ЛКК оформляється листок непрацездатності на 4 дні A sick leave for 4 days is issued through the LKK

Через ЛКК оформляється листок непрацездатності на 28 днів A sick leave for 28 days is issued through the LKK

86 / 200
Для визначення впливу мікроклімату на функціональний стан організму досліджувались наступні фізіологічні показники напруження функцій органів і систем, які беруть участь в теплообмінних процесах: пульс, артеріальний тиск, кількість дихань за хвилину, температура шкіри, величина потовиділення за хвилину, швидкість зорової та слухової реакцій. Назвіть показник, що найбільш об’єктивно відображає напруження процесів терморегуляцій організму: To determine the influence of the microclimate on the functional state of the body, the following physiological indicators of the stress on the functions of organs and systems involved in heat exchange processes were studied: pulse, blood pressure, number of breaths per minute, skin temperature, amount of perspiration per minute, speed of visual and auditory reactions. Name the indicator that most objectively reflects the stress of the body's thermoregulation processes:

Швидкість зорової та слухової реакції Speed of visual and auditory reaction

Потовиділення за хвилину Sweat per minute

Кількість дихальних актів за хвилину Number of breaths per minute

Пульс і артеріальний тиск Pulse and blood pressure

Температура шкіри Skin Temperature

87 / 200
Жінка 22-х років звернулася до жіночої консультації з приводу вагітності 11-12 тижнів. Під час обстеження виявлено позитивну реакцію Васермана. Дерматолог діагностував вторинний латентний сифіліс. Яка тактика ведення даної вагітності? A 22-year-old woman turned to a women's consultation about a pregnancy of 11-12 weeks. During the examination, a positive Wasserman reaction was found. The dermatologist diagnosed secondary latent syphilis. What tactics management of this pregnancy?

Протисифілітичне лікування тричі протягом вагітності Antisyphilitic treatment three times during pregnancy

Пролонгування вагітності після першого курсу протисифілітичної терапії Prolongation of pregnancy after the first course of antisyphilitic therapy

Термінове переривання вагітності Emergency termination of pregnancy

Штучне переривання вагітності до курсу протисифілітичної терапії Artificial termination of pregnancy before the course of antisyphilitic therapy

Штучне переривання вагітності після курсу протисифілітичної терапії Artificial termination of pregnancy after a course of antisyphilitic therapy

88 / 200
На північній околиці населеного пункту розташований хімічний комбінат. Протягом року спостерігається така повторюваність вітрів: північний -10%, східний - 20%, південний - 50%, західний - 20%. Назвіть оптимальне місце для розміщення земельної ділянки стаціонару медико-санітарної частини: A chemical plant is located on the northern outskirts of the settlement. During the year, the following recurrence of winds is observed: north -10%, east - 20%, south - 50%, west - 20%. Name the optimal place for placing the land plot of the hospital of the medical and sanitary department:

На південь від хімічного комбінату South of the chemical plant

На захід від хімічного комбінату West of the chemical plant

На північ від хімічного комбінату North of the chemical plant

На території хімічного комбінату On the territory of the chemical plant

На схід від хімічного комбінату East of the chemical plant

89 / 200
Жінка 36-ти років звернулась до жіночої консультації із скаргами на рясні болючі менструації, 'мажучі', шоколадного кольору виділення із піхви до та після менструації. В анамнезі 4 аборта. Гінекологічний статус: зовнішні статеві органи, шийка матки та піхва не змінені, матка у ретрофлексії, розмірами 9х7х8 см, обмежено рухома. Додатки не змінені, склепіння піхви вільні. Який діагноз? A 36-year-old woman turned to a gynecological clinic with complaints of heavy painful menstruation, 'smelling', chocolate-colored discharge from the vagina before and after menstruation. History 4 abortion. Gynecological status: external genitalia, cervix and vagina unchanged, uterus in retroflexion, 9x7x8 cm in size, limited mobility. Appendages unchanged, vaginal vault free. What is the diagnosis?

Хронічний ендометрит Chronic endometritis

Міома матки Uterine myoma

Ендометріоз тіла матки Endometriosis of the uterine body

Дисфункціональна кровотеча Dysfunctional bleeding

Рак ендометрію Endometrial cancer

90 / 200
У хлопчика 14-ти років із загостренням вторинного обструктивного пієлонефриту із сечі виділена синьогнійна паличка в титрі 1000000 мікробних тіл на 1 мл. Який антибактеріальний препарат найбільш доцільно призначити в даному випадку? A 14-year-old boy with an exacerbation of secondary obstructive pyelonephritis was isolated from the urine of Pseudomonas aeruginosa with a titer of 1,000,000 microbial bodies per 1 ml. Which antibacterial drug is the most appropriate to prescribe in this case case?

Азітроміцин Azithromycin

Цефазолін Cefazolin

Левоміцетин Levomycetin

Ципрофлоксацин Ciprofloxacin

Ампіцилін Ampicillin

91 / 200
Хвора 18-ти років звернулася до гінеколога зі скаргами на появу бородавчастих підвищень у ділянці статевих органів. Під час огляду зовнішніх статевих органів на великих і малих статевих губах визначаються сосочкоподібні розростання, м’якої консистенції, безболісні. При гінекологічному дослідженні патології з боку внутрішніх статевих органів не виявлено. Який попередній діагноз? An 18-year-old patient turned to a gynecologist with complaints about the appearance of warty growths in the area of the genitals. During the examination of the external genitalia, papillomas are identified on the labia majora and labia minora growths, soft consistency, painless. Gynecological examination revealed no pathology of the internal genital organs. What is the previous diagnosis?

Гострокінцеві кондиломи Condyloma acuminate

Рак вульви Vulvar cancer

Сифілітичні кондиломи Syphilitic condylomas

Папіломатоз Papillomatosis

Вегетуюча пухирчатка Vegetable pemphigus

92 / 200
У хлопчика 14-ти років на тлі хронічного тонзиліту та гаймориту з’явились відчуття перебоїв у роботі серця і додаткових пульсових ударів. ЧСС- 83/хв. На ЕКГ: після кожних двох синусових скорочень регулярно виникають імпульси, в яких відсутній зубець Р, QRS поширений більше 0,11 с, різко деформований, дискордантний зубець Т, після чого реєструється повна компенсаторна пауза. Вкажіть характер порушень ритму: A 14-year-old boy, against the background of chronic tonsillitis and sinusitis, felt interruptions in the work of the heart and additional pulse beats. Heart rate - 83/min. On the ECG : after every two sinus contractions, there are regular impulses in which there is no P wave, QRS is extended for more than 0.11 s, sharply deformed, discordant T wave, after which a complete compensatory pause is registered. Specify the nature of rhythm disturbances:

Часткова АV-блокада Partial AV block

Повна АV-блокада Complete AV block

Блокада лівої ніжки пучка Гіса Blockade of the left leg of the bundle of His

Екстрасистолія за типом тригемінії Extrasystole according to the type of trigeminy

Екстрасистолія за типом бігемінії Extrasystole according to the type of bigeminia

93 / 200
У дитини 10-ти років з неревматичним кардитом періодично виникають напади, які проявляються відчуттям болю в ділянці серця, задишкою, блідістю, підвищенням артеріального тиску, різким збільшенням ЧСС до 180/хв. Який з медикаментозних засобів є найбільш ефективним у лікуванні? A 10-year-old child with non-rheumatic carditis periodically has attacks, which are manifested by a feeling of pain in the heart area, shortness of breath, pallor, an increase in blood pressure, a sharp increase in heart rate to 180/min. Which medication is the most effective in treatment?

Лідокаїн Lidocaine

Аймалін Aimalin

Новокаїнамід Novocaineamide

Верапаміл Verapamil

Обзидан Obzydan

94 / 200
Дворічна дитина кашляє близько місяця. Протягом останніх 3 тижнів кашель став нападоподібним. Після нападу кашлю - блювання. Об’єктивно: одутлість обличчя. Субкон’юнктивальний крововилив справа. На шкірі шиї та грудної клітки поодинокі петехії, виразка на вуздечці язика. Який найбільш імовірний діагноз? A two-year-old child has been coughing for about a month. Over the past 3 weeks, the cough has become paroxysmal. After the coughing attack, vomiting. Objectively: swelling of the face. Subconjunctival hemorrhage on the right. Single petechiae on the skin of the neck and chest, an ulcer on the frenulum of the tongue. What is the most likely diagnosis?

Спастичний бронхіт Spastic bronchitis

Коклюш Whooping cough

РС-інфекція MS infection

Геморагічний васкуліт Hemorrhagic vasculitis

Туберкульозний бронхоаденіт Tuberculous bronchoadenitis

95 / 200
До гінекологічного стаціонару звернулася жінка 36-ти років зі скаргами на значну кровотечу зі статевих шляхів та затримку місячних на місяць. При бімануальному дослідженні: шийка матки бочкоподібної форми, м’якої консистенції. Матка звичайних розмірів, дещо розм’якшена. Придатки без особливостей з обох сторін. При дзеркальному дослідженні: шийка матки синюшна, збільшена у розмірах, зовнішнє вічко розкрите до 0,5 см. Дослідження сечі на ХГ -позитивне. Який найбільш імовірний діагноз? A 36-year-old woman came to the gynecological hospital with complaints of significant bleeding from the genital tract and a delay of menstruation for a month. On bimanual examination: the cervix is barrel-shaped, m 'what consistency. Uterus of normal size, somewhat softened. Appendages without features on both sides. On endoscopy: the cervix is bluish, enlarged, the external eye is open up to 0.5 cm. Urine test for HCG is positive. Which is the most probable diagnosis?

Загроза переривання вагітності Threat of abortion

Аборт у ходу Abort in progress

Маткова вагітність Uterine pregnancy

Позаматкова вагітність Ectopic pregnancy

Шийкова вагітність Cervical pregnancy

96 / 200
Доношена дитина, народилась з масою тіла 3200 г, довжиною тіла 50 см, з оцінкою за шкалою Апгар 8-10 балів. Який оптимальний строк першого прикладання до грудей? A full-term child, born with a body weight of 3200 g, a body length of 50 cm, with an Apgar score of 8-10 points. What is the optimal time for the first application to the breast?

В перші 48 годин In the first 48 hours

В перші 24 години In the first 24 hours

Після 48 годин After 48 hours

В перші 30 хвилин In the first 30 minutes

В перші 6 годин In the first 6 hours

97 / 200
Після бесіди з матір’ю семимісячного хлопчика, який знаходиться на природному вигодовуванні, педіатр з’ясував, що годують дитину 7 разів на добу. Яка кількість годувань установлена для дитини даного віку? After a conversation with the mother of a seven-month-old boy who is on natural feeding, the pediatrician found out that the child is fed 7 times a day. What is the number of feedings set for of a child of this age?

6 разів 6 times

3 рази 3 times

4 рази 4 times

7 разів 7 times

5 разів 5 times

98 / 200
Хлопчик 12-ти років скаржиться на 'голодні' нічні болі у животі, закрепи, печію. Хворіє більше двох років. Яке захворювання найбільш імовірне? A 12-year-old boy complains of 'hungry' night stomach pains, constipation, heartburn. He has been sick for more than two years. What disease is most likely?

Гострий гастрит Acute gastritis

Хронічний гастрит Chronic gastritis

Виразкова хвороба шлунка Gastric ulcer

Виразкова хвороба 12-палої кишки Duodenal ulcer disease

Дискінезія жовчовивідних шляхів Biliary tract dyskinesia

99 / 200
Хвора 40-ка років хворіє на ревматизм. Скаржиться на сильний біль у лівому оці, особливо вночі, погіршення зору, світлобоязнь, сльозотечу. Захворювання ні з чим не пов’язує. Об’єктивно: слабка перикорнеальна ін’єкція, згладженість рельєфу райдужки, зміна її кольору. Який найбільш імовірний діагноз? A 40-year-old patient suffers from rheumatism. She complains of severe pain in the left eye, especially at night, visual impairment, photophobia, lacrimation. The disease is unrelated It hurts. Objectively: weak pericorneal injection, smoothing of the relief of the iris, change in its color. What is the most likely diagnosis?

Гострий напад глаукоми Acute attack of glaucoma

Хоріоїдит Choroiditis

Ірит Irit

Кератит Keratitis

Іридоцикліт Iridocyclite

100 / 200
Хворий 67-ми років скаржиться на задишку, біль у грудях, загальну слабкість. Хворіє 5 місяців. Об’єктивно: t°- 37,3°С, Рs-96/хв. Над правою легенею голосове тремтіння не визначається, перкуторний звук тупий, дихання не прослуховується. У харкотинні - домішок крові дифузно змішаної зі слизом. Який найбільш вірогідний діагноз? A 67-year-old patient complains of shortness of breath, chest pain, general weakness. He has been sick for 5 months. Objectively: t°- 37.3°С, Ps-96/min. Over the right lung, the voice tremor is not determined, the percussion sound is dull, breathing is not heard. In the sputum, there is an admixture of blood diffusely mixed with mucus. What is the most likely diagnosis?

Бронхоектатична хвороба Bronchoectatic disease

Великовогнищева пневмонія Large focus pneumonia

Ексудативний плеврит Exudative pleurisy

Рак легень Lung cancer

Вогнищевий туберкульоз легень Focal pulmonary tuberculosis

101 / 200
Прикарпатський регіон характеризується постійною високою (більше 80%) вологістю атмосферного повітря. В холодний період року при помірно низьких температурах повітря населення цього регіону відчуває сильний холод. Це зумовлено збільшенням віддачі тепла шляхом: The Carpathian region is characterized by constant high (more than 80%) atmospheric air humidity. In the cold period of the year, with moderately low air temperatures, the population of this region feels severe cold. This is due to an increase heat return by:

Радіації Radiation

Випромінювання Radiation

Конвекції Convections

Випаровування Evaporation

Кондукції Conducts

102 / 200
Недостатньо очищені відходи промислового підприємства скидаються у річку, вода з якої використовується для господарсько-питного водопостачання. Це спричиняє загибель деяких мікроорганізмів, порушення процесів самоочищення води та погіршення її якості, що може мати негативний вплив на стан здоров’я людей. Така дія факторів навколишнього середовища називається: Insufficiently purified waste from an industrial enterprise is discharged into a river, the water from which is used for domestic and drinking water supply. This causes the death of some microorganisms, disruption of water self-purification processes and deterioration of its quality, which can have a negative impact on people's health. This effect of environmental factors is called:

Комплексна Comprehensive

Комбінована Combined

Поєднана Combined

Непряма Indirect

Пряма Direct

103 / 200
Холодний період року характеризувався низькою температурою повітря, постійною хмарністю, снігопадами. У зв’язку з цим, перебування дітей дошкільної установи на відкритому повітрі було обмежено. При медичному огляді діти скаржились на сонливість, кволість, поганий апетит, спостерігалась низька активність на заняттях з фізичного виховання. При лабораторному дослідженні крові виявлено порушення фосфорнокальцієвого обміну. Причиною такого стану дітей може бути: The cold period of the year was characterized by low air temperature, constant cloudiness, and snowfall. In this regard, the children of the preschool institution were limited in their outdoor time. During the medical examination children complained of sleepiness, frailty, poor appetite, low activity in physical education classes was observed. A laboratory blood test revealed a violation of phosphorus-calcium metabolism. The reason for this condition of children may be:

Охолоджуючий мікроклімат в приміщені Cooling microclimate in the room

Надмірне фізичне навантаження Excessive physical activity

Нагріваючий мікроклімат в приміщені Heating microclimate in the room

Дискомфортний мікроклімат в приміщені Uncomfortable microclimate in the room

Ультрафіолетова недостатність Ultraviolet deficiency

104 / 200
Для боротьби з бур’янами на сільськогосподарських угіддях тривалий час застосовували гербіциди, які за ступенем стійкості у навколишньому середовищі відносяться до стійких. Укажіть найбільш імовірний шлях надходження їх з ґрунту в організм людини: For a long time, herbicides have been used to control weeds on agricultural land, which are persistent in terms of their persistence in the environment. Specify the most likely route of their entry from the soil into the human body:

Ґрунт-найпростіші-людина Soil-protozoa-man

Ґрунт-комахи-людина Soil-Insect-Human

Ґрунт-рослини-людина Soil-Plant-Human

Ґрунт-тварини-людина Soil-animal-human

Ґрунт-мікроорганізми-людина Soil-Microorganisms-Human

105 / 200
Хворий 65-ти років тиждень тому переніс гострий інфаркт міокарда. Загальний стан погіршився, турбують задуха в спокої, виражена слабкість, наявність набряків та асциту. Межі серця розширені, відзначається парадоксальна перикардіальна пульсація латеральніше від верхівкового поштовху зліва. Який найбільш вірогідний діагноз? A 65-year-old patient suffered an acute myocardial infarction a week ago. His general condition has worsened, he is troubled by shortness of breath at rest, marked weakness, the presence of edema and ascites. The borders of the heart are enlarged, there is a paradoxical pericardial pulsation lateral to the apical impulse on the left. What is the most likely diagnosis?

Кардіосклеротична аневризма серця Cardiosclerotic aneurysm of the heart

Гострий перикардит Acute pericarditis

Хронічна аневризма серця Chronic heart aneurysm

Гостра аневризма серця Acute heart aneurysm

Ревматична аневризма серця Rheumatic heart aneurysm

106 / 200
Хвора 51-го року скаржиться на значні кров’янисті виділення із піхви протягом 15-ти діб. Із анамнезу: порушення менструальної функції впродовж року, хвора відмічає підвищену дратівливість, порушення сну. УЗД: матка відповідає віковим нормам, додатки без особливостей, товщина ендометрію 14 мм. Яка тактика лікаря? A 51-year-old patient complains of significant vaginal bleeding for 15 days. From the anamnesis: menstrual dysfunction for a year, the patient notes increased irritability , sleep disturbance. Ultrasound: the uterus corresponds to age norms, the appendages are without features, the thickness of the endometrium is 14 mm. What are the doctor's tactics?

Діагностичне вишкрібання стінок порожнини матки Diagnostic scraping of the walls of the uterine cavity

Надпіхвова ампутація матки без додатків Supravaginal amputation of uterus without attachments

Обстеження на TORCH-iнфекцiю Examination for TORCH-infection

Консервативне лікування кровотечі Conservative treatment of bleeding

Гістеректомія Hysterectomy

107 / 200
Хвора 18-ти років скаржиться на болючість і нагрубання молочних залоз, головний біль, дратівливість, набряклість нижніх кінцівок. Дані симптоми турбують з початку менархе, з’являються за 3-4 дні до початку чергової менструації. При гінекологічному огляді патології не виявлено. Яке захворювання у жінки? An 18-year-old patient complains of soreness and engorgement of the mammary glands, headache, irritability, swelling of the lower extremities. These symptoms bother her from the beginning of menarche, appear after 3-4 days before the start of the next menstruation. During a gynecological examination, no pathology was detected. What disease does the woman have?

Захворювання нирок Kidney disease

Захворювання серцево-судинної системи Diseases of the cardiovascular system

Мастопатія Mastopathy

Передменструальний синдром Premenstrual syndrome

Неврастенія Neurasthenia

108 / 200
Хвора 52-х років скаржиться на біль у правій половині грудної клітини, задишку, кашель з великою кількістю пінистого харкотиння у вигляді 'м’ясних помиїв', з неприємним запахом. Об’єктивно: стан тяжкий, ціаноз, ЧДР- 31/хв, при перкусії вкорочений перкуторний звук над правою легенею, аускультативно - різнокаліберні вологі хрипи. Який найбільш вірогідний діагноз? A 52-year-old patient complains of pain in the right half of the chest, shortness of breath, cough with a lot of foamy sputum in the form of 'meat slops', with unpleasant by smell. Objectively: the condition is severe, cyanosis, CHDR - 31/min, on percussion, a shortened percussion sound over the right lung, auscultation - wet rales of various calibers. What is the most likely diagnosis?

Гангрена легені Gangrene of the lungs

Емпієма плеври Empyema of the pleura

Хронічна пневмонія Chronic pneumonia

Бронхоектатична хвороба Bronchoectatic disease

Абсцес легені Lung abscess

109 / 200
Хвора 23-х років доставлена в клініку після автодорожньої травми. Стан хворої тяжкий, у свідомості, шкірні покрови бліді, Рs- 100/хв., АТ- 90/60 мм рт.ст., ЧД- 29/хв., справа дихання не вислуховується, визначається виражена підшкірна емфізема. Рентгенологічно - права легеня колабована, зламані V-VІ ребра справа. Яка тактика лікаря? A 23-year-old female patient was brought to the clinic after a road traffic injury. The patient's condition is serious, she is conscious, her skin is pale, Ps- 100/min., BP- 90 /60 mm Hg, BH- 29/min., breathing is not heard on the right, marked subcutaneous emphysema is determined. X-ray - the right lung is collapsed, the V-VI ribs on the right are broken. What are the doctor's tactics?

Дренування плевральної порожнини Drainage of the pleural cavity

Виконання бронхоскопії Performance of bronchoscopy

Термінова торакотомія з ушиванням пошкодженої легені Urgent thoracotomy with suturing of the damaged lung

Проведення протишокової терапії Conducting anti-shock therapy

Виконання міжреберної спирт-новокаїнової блокади Performance of intercostal alcohol-novocaine blockade

110 / 200
Хворий 15-ти років відстає у фізичному розвитку, періодичне пожовтіння шкіри. Об’єктивно: селезінка 16х12х10 см, холеци-столітіаз, виразка шкіри нижньої третини лівої гомілки. У крові: ер.- 3,0 • 1012/л, НЬ-90 г/л, КП-1,0, мікросфероцитоз, ретикулоцитоз. Білірубін крові 56 мкмоль/л, непрямий - 38 мкмоль/л. Оберіть метод лікування: A 15-year-old patient lags behind in physical development, periodic yellowing of the skin. Objectively: spleen 16x12x10 cm, cholecystolithiasis, skin ulcer of the lower third of the left leg. In the blood: ER - 3.0 • 1012/l, Hb-90 g/l, KP-1.0, microspherocytosis, reticulocytosis. Blood bilirubin 56 μmol/l, indirect - 38 μmol/l. Choose the method of treatment:

Спленектомія Splenectomy

Пересадка селезінки Spleen transplant

Оментоспленопексія Omentosplenopexy

Портокавальний анастомоз Portocaval anastomosis

Оментогепатопексія Omentohepatopexy

111 / 200
Дівчинці 8 місяців; народилася недоношеною. Під час огляду: відмічається задуха, тахікардія, гепатоспленомегалія, відставання в фізичному розвитку, ціаноз кінцівок. Визначається парастернальний серцевий горб, у ІІ міжребер’ї зліва вислуховується систолодіастолічний шум, АТ- 90/0 мм рт.ст. Про яке захворювання слід думати? The girl is 8 months old; she was born prematurely. During the examination: shortness of breath, tachycardia, hepatosplenomegaly, delayed physical development, cyanosis of the extremities are noted. A parasternal cardiac hump is detected, in II a systolic diastolic murmur is heard in the left intercostal space, blood pressure is 90/0 mm Hg. What disease should we think about?

Незарощення міжшлуночкової перегородки Non-occlusion of the interventricular septum

Коарктація аорти Coarctation of the aorta

Стеноз легеневої артерії Pulmonary artery stenosis

Стеноз аортального клапана Aortic valve stenosis

Відкрита артеріальна протока Open ductus arteriosus

112 / 200
Лікар швидкої допомоги приїхав на виклик до пацієнтки 76-ти років, яка впала на праву ногу. Скарги на біль в ділянці правого кульшового суглоба. Порушення опорної функції правої нижньої кінцівки. Об’єктивно: права нижня кінцівка в положенні зовнішньої ротації, позитивний симптом 'прилиплої п’ятки', біль при навантаженні по вісі на великий вертлюг. Який найбільш імовірний діагноз? The emergency doctor came on call to a 76-year-old female patient who fell on her right leg. Complaints of pain in the area of the right hip joint. Violation of the supporting function of the right lower limbs. Objectively: the right lower extremity in the position of external rotation, a positive symptom of a 'sticky heel', pain with axial load on the large acetabulum. What is the most likely diagnosis?

Задньо-нижній вивих стегна Posterior-inferior hip dislocation

Перлом верхньої третини правого стегна Pearl of the upper third of the right thigh

Перелом шийки правого стегна Fracture of the neck of the right femur

Забій правого кульшового суглоба Contusion of the right hip joint

Правобічний коксартроз Right-sided coxarthrosis

113 / 200
З відмороженнями обох ступней хворий доставлений до приймального відділення. Що необхідно зробити потерпілому? With frostbite on both feet, the patient was taken to the reception department. What should be done to the victim?

Помістити ноги в гарячу воду Put your feet in hot water

Накласти спиртовий компрес Apply an alcohol compress

Призначити серцеві препарати Prescribe cardiac drugs

Накласти пов’язку, ввести судиннорозширюючі препарати Apply a bandage, introduce vasodilator drugs

Розтерти снігом ступні Rubbing feet with snow

114 / 200
На четверту добу після отримання колотої рани правої стопи у хворого підвищилася температура тіла до 38oC, пахвинні лімфатичні вузли збільшились, стали болючі, шкіра над ними почервоніла. Про яке ускладнення рани можна думати? On the fourth day after receiving a puncture wound on the right foot, the patient's body temperature rose to 38oC, the inguinal lymph nodes increased in size, became painful, and the skin above them turned red. What a complication wounds can be thought?

Бешиха Beshikha

Лімфангоїт Lymphangitis

Флегмона Phlegmon

Правець Tetanus

Лімфаденіт Lymphadenitis

115 / 200
Хворий 50-ти років звернувся зі скаргами на підвищення температури тіла до 39°С, пульсуючий біль та припухлість правої кисті. Напередодні уколов кисть риб’ячою кісткою. При огляді відмічається набряк і різкий біль в долоні та в ділянці підвищення I пальця кисті. Окрім цього відзначається набряк м’яких тканин тилу кисті, почервоніння шкіри та різкий біль при рухах пальців. Яке захворювання у пацієнта? A 50-year-old patient complained of an increase in body temperature to 39°C, throbbing pain and swelling of the right hand. The day before, he pricked the hand with a fish bone. the examination shows swelling and sharp pain in the palm and in the area of the first finger of the hand. In addition, there is swelling of the soft tissues of the back of the hand, redness of the skin and sharp pain when moving the fingers. What disease does the patient have?

Сухожилковий панарицій I пальця кисті Tendinosus panaritium of the first finger of the hand

Флегмона правої кисті Phlegmon of the right hand

Стороннє тіло кисті Foreign body of the brush

Бешиха кисті Beshikha brush

Карбункул кисті Carbuncle of the hand

116 / 200
У місті на вибірковій сукупності вивчався вплив викидів у повітря відходів металургійного виробництва на захворюваність на обструктивний бронхіт. Розрахований коефіцієнт кореляції становив +0,79. Оцініть силу і напрямок зв’язку: In the city, the influence of emissions of metallurgical waste into the air on the incidence of obstructive bronchitis was studied on a sample population. The calculated correlation coefficient was +0.79. Estimate the strength and direction of the 'link:

- -

Зворотній, сильний Reverse, strong

Прямий, середній Direct, average

Прямий, сильний Direct, strong

Зворотній, середній Reverse, Average

117 / 200
До жіночої консультації звернулася жінка 22-х років на 15-ту добу після кесаревого розтину зі скаргами на біль у правій молочній залозі, підвищення температури тіла до 39°С, озноб. Об’єктивно: молочна залоза збільшена, гіперемована, ущільнена, болюча під час пальпації. Дитину годує груддю з 8-ї доби, молоко зціджує нерегулярно. Яка профілактика цього стану? A 22-year-old woman turned to the women's consultation on the 15th day after cesarean section with complaints of pain in the right mammary gland, an increase in body temperature to 39°C , chills. Objectively: the mammary gland is enlarged, hyperemic, compacted, painful during palpation. The child has been breastfed since the 8th day, milk is expressed irregularly. What is the prevention of this condition?

Регуляція менструального циклу Regulation of the menstrual cycle

Регулярне зціджування молочних залоз після годування, допологова підготовка сосків та молочних залоз Regular pumping of mammary glands after feeding, prenatal preparation of nipples and mammary glands

Зціджування обох молочних залоз після кожного годування Expressing both mammary glands after each feeding

Проведення профоглядів Professional reviews

Регулярні профогляди, використання ВМС Regular professional examinations, use of IUD

118 / 200
Службовець хворіє 4 місяці, необхідно подальше лікування, хворий непрацездатний. Хто уповноважений проводити експертизу непрацездатності даного хворого? The employee has been ill for 4 months, further treatment is required, the patient is incapacitated. Who is authorized to conduct an examination of the incapacity of this patient?

Лікарсько-консультативна комісія Medical Advisory Commission

Медико-соціальна експертна комісія Medical and social expert commission

Лікуючий лікар із завідувачем відділення Treating doctor with head of department

Головний лікар лікувально-профілактичного закладу Chief physician of the medical and preventive institution

Заступник головного лікаря по експертизі непрацездатності Deputy chief physician for disability examination

119 / 200
Впродовж тривалого часу проводиться вивчення тенденцій у змінах показників загальної смертності різних груп населення, які проживають на різних адміністративних територіях. Який із статистичних методів може бути використано з цією метою? Over a long period of time, trends in changes in total mortality rates of different population groups living in different administrative territories have been studied. Which of the statistical methods can be used for this purpose?'

Аналіз стандартизованих показників Analysis of standardized indicators

Аналіз динамічних рядів Analysis of dynamic series

Аналіз рівнів відносних величин Analysis of levels of relative values

Кореляційно-регресивний аналіз Correlation-regression analysis

Оцінка вірогідності різниці показників Estimation of the probability of the difference of indicators

120 / 200
Хвора 73-х років надійшла в клініку в екстреному порядку із защемленою стегновою килою. Крім того у хворої наявна варикозна хвороба лівої нижньої кінцівки, ожиріння IV ст. Яке найбільш імовірне ускладнення може виникнути у післяопераційному періоді? A 73-year-old patient came to the clinic as an emergency with a pinched femoral hernia. In addition, the patient has varicose disease of the left lower limb, IV degree obesity. What is the most a possible complication may occur in the postoperative period?

Інфаркт міокарда Myocardial infarction

Гостра дихальна недостатність Acute respiratory failure

Тромбоемболія легеневої артерії Thromboembolism of the pulmonary artery

Сепсис Sepsis

Гостра пневмонія Acute pneumonia

121 / 200
Хворий 28-ми років поступив у відділення інтенсивної терапії зі скаргами на болі в животі, блювання, двоїння в очах. Об’єктивно: дезорієнтований, зіниці розширені, реакція на світло ослаблена, шкіра суха, гіперемована, незначний ціаноз, АТ- 90/60 мм рт.ст., РS100/хв. З анамнезу відомо, що за два дні до госпіталізації вживав алкоголь невідомого походження. Специфічна терапія полягає у внутрішньовенному введенні: A 28-year-old patient was admitted to the intensive care unit with complaints of abdominal pain, vomiting, double vision. Objectively: disoriented, dilated pupils, reaction weak to light, skin dry, hyperemic, slight cyanosis, blood pressure - 90/60 mm Hg, blood pressure 100/min. It is known from the anamnesis that two days before hospitalization he consumed alcohol of unknown origin. The specific therapy consists in intravenous administration:

40% розчину глюкози 40% glucose solution

Гідрокарбонату натрію Sodium bicarbonate

Тіаміну Thiamine

Есенціале Essentially

Етанолу Ethanol

122 / 200
У хворої 50-ти років впродовж півтора місяця відзначається стабільне наростання жовтяниці, анемії з періодичними підйомами температури тіла. Під час пальпації виявлено збільшений і безболісний жовчний міхур. Розвиток якого захворювання можна припустити? In a 50-year-old patient, a stable increase in jaundice and anemia with periodic rises in body temperature was noted for one and a half months. During palpation, an enlarged and painless gallbladder was found. The development of which can the disease be assumed?

Первинний склерозуючий холангіт Primary sclerosing cholangitis

Рак жовчного міхура Gall bladder cancer

Рак головки підшлункової залози Cancer of the head of the pancreas

Жовчнокам’яна хвороба Cholelithiasis

Ехінококоз печінки Echinococcosis of the liver

123 / 200
Вагітна з терміном гестації 7 тижнів поступила у пологовий будинок у важкому стані зі скаргами на блювання до 20 разів на добу, слабкість, запаморочення, відразу до їжі. За вагітність маса зменшилася на 10 кг. Рs-105/хв., ритмічний, АТ- 90/60 мм рт.ст., температура тіла 37,9°С. У крові: Нb- 154 г/л, у сечі ацетон (++++). Комплексна терапія, що проводиться, неефективна. Яка акушерська тактика? A pregnant woman with a gestation period of 7 weeks was admitted to the maternity ward in critical condition with complaints of vomiting up to 20 times a day, weakness, dizziness, aversion to food. For pregnancy weight decreased by 10 kg. Ps-105/min., rhythmic, BP- 90/60 mm Hg, body temperature 37.9°C. In blood: Hb- 154 g/l, in urine acetone (++ ++). The complex therapy being carried out is ineffective. What are the obstetric tactics?

Переривання вагітності Termination of pregnancy

Перевести хвору у гастроентерологічне відділення Transfer the patient to the gastroenterology department

Продовжити консервативну терапію гестозу Continue conservative therapy of preeclampsia

Продовжити лікування протягом 1 тижня, потім вирішити питання про можливість Continue treatment for 1 week, then decide on the possibility

Застосувати у лікуванні хворої плазмаферез Use plasmapheresis in the treatment of the patient

124 / 200
124. При терміні гестації 32 тижні у вагітної почались передчасні пологи. Перейми через 10-15 хвилин, по 15-20 секунд. Серцебиття плоду ясне, ритмічне - 145/хв. При піхвовому дослідженні шийка матки вкорочена, зовнішнє вічко розкрите на 1,5 см, плідний міхур цілий, передлежить голівка, виділення слизові, температура тіла 36,5°С. У крові: без відхилень. У мазку з піхви 4-5 лейкоцитів у п/з. Яка акушерська тактика? 124. At 32 weeks of gestation, a pregnant woman went into premature labor. Contractions after 10-15 minutes, 15-20 seconds each. The fetal heartbeat is clear, rhythmic - 145 /min. During vaginal examination, the cervix is shortened, the external eye is opened by 1.5 cm, the amniotic sac is intact, the head is present, mucous discharge, body temperature is 36.5°C. In the blood: no abnormalities. In the vaginal smear 4-5 leukocytes in pregnancy. What are the obstetric tactics?

Операція кесарського розтину Caesarean section operation

Пологи вести через природні пологові шляхи Give birth through the natural birth canal

Спазмолітики, анальгетики Spasmolytics, analgesics

Дати внутрішньовенний наркоз для зняття пологової діяльності Give intravenous anesthesia to relieve labor

Токоліз. Профілактика дистрес-синдрому плоду Tocolysis. Prevention of fetal distress syndrome

125 / 200
Цеховий лікар формує для поглибленого спостереження групу осіб, що часто хворіють. При цьому він бере до уваги кількість етіологічно пов’язаних випадків захворювань з тимчасовою втратою працездатності протягом останнього року у кожного із робітників. Якою повинна бути ця кількість, щоб працівника віднесли до вказаної групи? The shop doctor forms a group of frequently ill persons for in-depth observation. At the same time, he takes into account the number of etiologically related cases of diseases with temporary loss of working capacity during the last year for each of the workers. What should be this number in order for the worker to be assigned to the specified group?

6 і більше 6 or more

2 і більше 2 or more

3 і більше 3 or more

4 і більше 4 or more

1 і більше 1 or more

126 / 200
Хворий 53-х років поступив у клініку з районної лікарні, де 5 днів тому лінійним розрізом був розкритий карбункул спини. В післяопераційному періоді стан залишався важким. Констатовано тахіпноє, тахікардія, лейкоцитоз, гіпертермія. При бактеріологічному обстеженні крові виділений гемолітичний стрептокок. Який найбільш імовірний діагноз? A 53-year-old patient was admitted to the clinic from the district hospital, where 5 days ago a carbuncle of the back was opened by a linear incision. In the postoperative period, the condition remained severe. He was found to be tachypneic, tachycardia, leukocytosis, hyperthermia. Bacteriological examination of blood revealed hemolytic streptococcus. What is the most likely diagnosis?

Сепсис Sepsis

Токсикорезорбтивна гарячка Toxicoresorptive fever

Синдром системної запальної відповіді Systemic inflammatory response syndrome

Карбункул спини в стадії абцедування Back carbuncle in the stage of abscess

Септичний шок Septic shock

127 / 200
Вагітна 32-х років скаржиться на епізодичні втрати свідомості, спонтанні непритомності, що швидко зникають при зміні положення тіла. Непритомність може супроводжуватися брадикардією, що швидко проходить. У всьому іншому вагітність протікає без ускладнень. Назвіть найбільш імовірну причину подібного стану? A 32-year-old pregnant woman complains of episodic loss of consciousness, spontaneous fainting, which quickly disappear when changing the position of the body. Fainting may be accompanied by bradycardia, which quickly passes. In all another has an uncomplicated pregnancy. Name the most likely cause of such a condition?

Зменшення тиску у венах нижніх кінцівок Reduction of pressure in the veins of the lower extremities

Вегето-судинна дистонія за кардіальним типом Vegeto-vascular dystonia by cardiac type

Психо-соматичні порушення Psycho-somatic disorders

Стискання нижньої порожнистої вени вагітною маткою Compression of the inferior vena cava by the pregnant uterus

Підвищення тиску у венах верхніх кінцівок Increased pressure in the veins of the upper limbs

128 / 200
На 15-ту добу після пологів до гінеколога звернулась жінка зі скаргами на болі в правій молочній залозі, підвищення температури тіла до 38°С, слабкість. Об’єктивно: у верхньому квадранті правої молочної залози пальпується болючий інфільтрат 10х10 см, шкіра над ним гіперемована, з соска - гнійні виділення. Який найбільш імовірний діагноз? On the 15th day after giving birth, a woman turned to a gynecologist with complaints of pain in the right mammary gland, an increase in body temperature to 38°C, weakness. Objectively : a painful infiltrate measuring 10x10 cm is palpable in the upper quadrant of the right mammary gland, the skin above it is hyperemic, purulent discharge from the nipple. What is the most likely diagnosis?

Фіброзно-кістозна мастопатія Fibrocystic mastopathy

Мастопатія Mastopathy

Гнійний мастит Purulent mastitis

Лактостаз Lactostasis

Кіста правої молочної залози Right breast cyst

129 / 200
Для характеристики стану здоров’я населення в районі розраховувалися показники народжуваності, смертності, природного приросту, поширеності хвороб і первинної захворюваності, загальної інвалідності та інвалідизації населення. До якого виду статистичних величин відносяться ці показники? In order to characterize the state of health of the population in the district, indicators of birth rate, mortality, natural increase, prevalence of diseases and primary morbidity, general disability and disability of the population were calculated. To what type statistical values to which these indicators belong?

Наочності Visibility

Співвідношення Ratio

Інтенсивний Intensive

Стандартизований Standardized

Екстенсивний Extensive

130 / 200
Жінка 33-х років у минулому перенесла дві операції з приводу позаматкової вагітності, обидві маткові труби видалені. Звернулася на консультацію з питанням: що можна зробити, щоб настала вагітність? A 33-year-old woman underwent two operations for ectopic pregnancy in the past, both fallopian tubes were removed. She asked for a consultation with the question: what can be done to get pregnant ?

Індукція овуляції Ovulation induction

Штучне запліднення спермою донора Artificial insemination with donor sperm

Інсемінація спермою чоловіка Insemination with male sperm

Сурогатне материнство Surrogacy

Екстракорпоральне запліднення In vitro fertilization

131 / 200
Хвора 18-ти років через 2 тижні після ангіни скаржиться на підвищення температури тіла, болі й припухлість у колінних суглобах, висипку у вигляді червоних кілець на гомілках. Через кілька днів почав турбувати біль у гомілково-ступеневих, потім - у ліктях. Для якого захворювання характерні такі симптоми? 2 weeks after angina, an 18-year-old patient complains of an increase in body temperature, pain and swelling in the knee joints, a rash in the form of red rings on the lower legs. After a few days, he began to be bothered by pain in the ankles and feet, then in the elbows. What disease is characterized by such symptoms?

Деформуючий остеоартроз Deforming osteoarthritis

Токсико-алергічний дерматит Toxic-allergic dermatitis

Реактивний артрит Reactive arthritis

Ревматоїдний артрит Rheumatoid arthritis

Гостра ревматична лихоманка Acute rheumatic fever

132 / 200
У жінки 62-х років спостерігається слабкість, підвищена пітливість. В анамнезі: часті застудні захворювання. Об’єктивно: температура - 37,5°С. Пальпуються шийні лімфовузли розмірами до 1,5-2 см, безболісні під час пальпації, не спаяні, щільно-еластичної консистенції. В крові: лейкоцити - 30 • 109 /л, лімфоцити - 68%, пролімфоцити - 6%. Знайдені клітини лейколізу. Який з наступних діагнозів найбільш імовірний? A 62-year-old woman has weakness, increased sweating. History: frequent colds. Objectively: temperature - 37.5°C. Palpable cervical lymph nodes up to 1.5-2 cm in size, painless during palpation, not fused, dense-elastic consistency. In the blood: leukocytes - 30 • 109 /l, lymphocytes - 68%, prolymphocytes - 6%. Leukolysis cells were found. Which of the following which of the following diagnoses is the most probable?

Хронічний лімфолейкоз Chronic lymphocytic leukemia

Лімфогранулематоз Lymphogranulomatosis

Лімфома Lymphoma

Регіонарний лімфаденіт Regional lymphadenitis

Лімфосаркома Lymphosarcoma

133 / 200
У роділлі під час термінових пологів на тлі бурхливої пологової діяльності з’явився гострий біль у животі, припинилася пологова діяльність, почали наростати явища гіповолемічного шоку. Зовнішня кровотеча незначна. Який найбільш імовірний діагноз? In the delivery room, during urgent delivery, against the background of violent labor, acute abdominal pain appeared, labor stopped, hypovolemic shock began to increase. External bleeding is insignificant. What is the most likely diagnosis?

Передлежання плаценти Presentation of the placenta

Емболія навколоплідними водами Amniotic fluid embolism

Відшарування природно розташованої плаценти Detachment of naturally located placenta

Розрив матки Rupture of uterus

Вторинна слабкість пологової діяльності Secondary weakness of labor activity

134 / 200
Хворий на алкоголізм заявляє, що 'постійно чує за вікном чоловічі і жіночі голоси', які сперечаються між собою про нього: 'одні називають його алкашем, придурком, якого треба зарізати або повішати, інші - добрим татом, тому що він приніс дитині яблуко'. Про який психопатологічний синдром можна думати? A patient with alcoholism declares that he 'constantly hears male and female voices outside the window' arguing among themselves about him: 'some call him a drunkard, a jerk who must be slaughtered or hanged, others - a good dad, because he brought the child an apple'. What psychopathological syndrome can you think about?

Деліріозний Delirious

Параноїдний Paranoid

Галюцинаторний Hallucinatory

Абстинентний Abstinent

Онейроїдний Oneiroid

135 / 200
У коридорі лікарні хворому 55-ти років раптово стало погано, негайно оглянутий лікарем. При огляді: шкірні покриви бліді, самостійне дихання відсутнє, пульсу на сонних артеріях немає, зіниці не розширені. З якого заходу необхідно почати реанімацію, щоб відновити серцеву діяльність? In the corridor of the hospital, a 55-year-old patient suddenly felt ill, he was immediately examined by a doctor. On examination: the skin is pale, there is no independent breathing, there is no pulse on the carotid arteries, Pupils are not dilated. Which measure should be used to start resuscitation to restore cardiac activity?

Прекардіальний удар Precardiac attack

Дихання 'рот до роту' Mouth to mouth breathing

Зовнішній масаж серця External heart massage

Дефібриляція Defibrillation

Налагодження прохідності дихальних шляхів Adjustment of airway patency

136 / 200
У дівчинки 3-х місяців спостерігаються нежить, задишка, сухий кашель. Хворіє 2-гу добу. Об’єктивно: шкіра бліда, акроціаноз, дихання поверхневе, Ps- 80/хв., над усією поверхнею легень коробковий звук, велика кількість дрібнопухирцевих хрипів. Який найбільш імовірний діагноз? A 3-month-old girl has a runny nose, shortness of breath, dry cough. She has been ill for the 2nd day. Objectively: pale skin, acrocyanosis, shallow breathing, Ps - 80/min., over the entire surface of the lungs a box sound, a large number of small alveolar rales. What is the most likely diagnosis?

Муковісцидоз Cystic Fibrosis

Стороннє тіло дихальних шляхів Foreign body of respiratory tract

Гострий бронхіт Acute bronchitis

Пневмонія Pneumonia

Гострий бронхіоліт Acute bronchiolitis

137 / 200
Хвора 24-х років звернулася до клініки зі скаргами на збільшення маси тіла, підвищений апетит. Об’єктивно: гіперстенічної статури, індекс маси тіла 33,2 кг/м2 , обвід талії 100 см. Співвідношення обвіду талії до обвіду стегон 0,95. Який попередній діагноз? A 24-year-old patient came to the clinic with complaints of weight gain, increased appetite. Objectively: hypersthenic physique, body mass index 33.2 kg/ m2, waist circumference 100 cm. The ratio of waist circumference to hip circumference is 0.95. What is the previous diagnosis?

Аліментарно-конституційне ожиріння, I ст., абдомінальний тип Alimentary and constitutional obesity, 1st century, abdominal type

Аліментарно-конституційне ожиріння, III ст., геноїдний тип Alimentary and constitutional obesity, 3rd century, genoid type

Гіпоталамічне ожиріння за типом Іценко-Кушинга, I ст., абдомінальний тип Hypothalamic obesity according to Itsenko-Cushing type, 1st century, abdominal type

Гіпоталамічне ожиріння за типом Іценко-Кушинга, II ст., геноїдний тип Hypothalamic obesity according to Itsenko-Cushing type, II century, genoid type

Аліментарно-конституційне ожиріння, II ст., абдомінальний тип Alimentary and constitutional obesity, II century, abdominal type

138 / 200
Хворого 47-ми років почав турбувати стискаючий біль за грудниною, що виникає під час ходи на 700-800 м. Один раз на тиждень випиває 2 л пива. Артеріальна гіпертензія впродовж останніх 7-ми років. Об’єктивно: Ps- 74/хв., АТ- 120/80 мм рт.ст. При проведенні ВЕМ на навантаженні 75 Вт зареєстровано депресію сегмента ST на 2 мм нижче ізолінії у V4 — V6. Який найбільш імовірний діагноз? A 47-year-old patient began to be bothered by squeezing pain behind the sternum, which occurs during a walk of 700-800 m. He drinks 2 liters of beer once a week. Arterial hypertension during the last 7 years. Objectively: Ps - 74/min., BP - 120/80 mm Hg. When carrying out VEM at a load of 75 W, ST segment depression was registered 2 mm below the isoline in V4 — V6. What is the most likely diagnosis?

Стенокардія напруги, II функціональний клас Tension angina, II functional class

Алкогольна кардіоміопатія Alcoholic cardiomyopathy

Стенокардія напруги, IV функціональний клас Stress angina, IV functional class

Стенокардія напруги, III функціональний клас Stress angina, III functional class

Вегето-судинна дистонія за гіпертонічним типом Vegetovascular dystonia of the hypertensive type

139 / 200
33-х років гостра крововтрата (ер.- 2,2 • 1012 139. /л, НЬ- 55 г/л), група крові А(ІІ)КН+. Йому помилково проведено трансфузію донорської еритромаси АВ(IV)Rh+. Через годину з’явилися відчуття тривоги, біль у попереку, животі. Рs-134/хв., АТ- 100/65 мм рт.ст., температура тіла - 38,6°С. При катетеризації сечового міхура отримано 12 мл/год сечі темнокоричневого кольору. Яке ускладнення виникло у хворого? 33-year-old acute blood loss (ER - 2.2 • 1012 139. /l, Hb - 55 g/l), blood group A (II )KN+. He was mistakenly transfused with donor erythromass AB(IV)Rh+. An hour later, feelings of anxiety appeared, pain in the lower back, abdomen. Ps-134/min., BP- 100/65 mm Hg, body temperature - 38.6°C. During bladder catheterization, 12 ml/h of dark brown urine was obtained. What complication did the patient experience?

Гостра ниркова недостатність Acute renal failure

Іфекційно-токсичний шок Infectious-toxic shock

Цитратна інтоксикація Citrate intoxication

Кардіальний шок Cardiac shock

Алергічна реакція на донорську еритро-масу Allergic reaction to donor erythrocyte mass

140 / 200
У жінки 67-ми років, що страждає на гіпертонічну хворобу, вночі раптово з’явилися головний біль, задишка, що швидко перейшла в ядуху. Об’єктивно: бліда, на лобі краплі поту, АТ- 210/140 мм рт.ст., ЧСС-120/хв., над легенями поодинокі сухі хрипи, в нижніх відділах - вологі хрипи. Гомілки пастозні. Яка невідкладна допомога найбільш доцільна у даному випадку? A 67-year-old woman suffering from hypertension suddenly developed a headache and shortness of breath during the night, which quickly turned into dyspnea. Objectively: pale, drops of sweat on the forehead, blood pressure - 210/140 mm Hg, heart rate - 120/min, single dry rales above the lungs, wet rales in the lower parts. The shins are pasty. What emergency aid is most appropriate in this case?

Еналаприл та фуросемід в/в Enalapril and Furosemide IV

Дігоксин та нітрогліцерин в/в Digoxin and IV nitroglycerin

Лабеталол та фуросемід в/в Labetalol and furosemide IV

Нітрогліцерин та фуросемід в/в Nitroglycerin and Furosemide IV

Нітрогліцерин в/в та капотен всередину Nitroglycerin intravenously and capoten internally

141 / 200
В інфекційний стаціонар госпіталізована хвора 35-ти років зі скаргами на остуду, підвищення температури до 39°С, нудоту, багаторазове блювання, ниючий біль у животі, часті рідкі випорожнення з неприємним запахом, зеленого кольору до 10-12 раз на добу. Шкірні покриви бліді, тургор тканин дещо знижений, живіт м’який, болючий у мезогастрії, бурчання в правій здухвинній ділянці. Який найбільш імовірний діагноз? A 35-year-old patient was hospitalized in an infectious disease hospital with complaints of a cold, temperature rise to 39°C, nausea, repeated vomiting, aching abdominal pain, frequent stools with an unpleasant smell, green in color up to 10-12 times a day. The skin is pale, tissue turgor is slightly reduced, the abdomen is soft, painful in the mesogastric area, grumbling in the right sphincter area. What is the most likely diagnosis?

Холера Cholera

Харчова токсикоінфекція Food poisoning

Сальмонельоз Salmonellosis

Шигельоз Shigelosis

Ротавірусна інфекція Rotavirus infection

142 / 200
Хворий 42-х років госпіталізований зі скаргами на головний біль, висип на тілі. Під час огляду: t°- 40°С, збуджений, марить, обличчя гіперемоване, язик збільшений, висувається поштовхоподібно, петехіальна висипка на тулубі, кінцівках, тахікардія, гіпотонія, гепатоспленомегалія. Немає постійного місця проживання, безробітний. Який найбільш імовірний діагноз? A 42-year-old patient was hospitalized with complaints of a headache, a rash on the body. During the examination: t°- 40°С, excited, delirious, hyperemic face , the tongue is enlarged, protrudes jerkily, petechial rash on the trunk, limbs, tachycardia, hypotension, hepatosplenomegaly. No permanent place of residence, unemployed. What is the most likely diagnosis?

Черевний тиф Typhoid

Лептоспіроз Leptospirosis

ВІЛ-інфекція HIV infection

Грип Flu

Висипний тиф Typhoid

143 / 200
Хворий на гастроезофагеальну рефлюксну хворобу впродовж 5-ти років періодично, за рекомендацією фармацевта, приймає препарат 'що знижує кислотність'. Виникли наступні побічні ефекти: остеопороз, м’язова слабкість, нездужання. Який препарат має такий побічний ефект? A patient with gastroesophageal reflux disease has been periodically taking an acid-reducing drug for 5 years, on the recommendation of a pharmacist. The following side effects have occurred: osteoporosis, m' ulcer weakness, malaise. What drug has such a side effect?

β2-блокатор β2-blocker

Алюмінійвміщуючий антацид Aluminium-containing antacid

Інгібітор протонної помпи Proton pump inhibitor

Гастроцепін Gastrotsepin

Метоклопрамід Metoclopramide

144 / 200
Хворий 44-х років під час сільськогосподарських робіт отримав рану в ділянці правої гомілки. Профілактичного щеплення проти правця не отримував протягом останніх 20-ти років. Яку профілактику правця слід провести? A 44-year-old patient received a wound in the right lower leg during agricultural work. He did not receive a preventive vaccination against tetanus for the past 20 years. What tetanus prevention should be done conduct?

Виконати первинну хірургічну обробку рани Perform initial surgical treatment of the wound

Специфічна - активно-пасивна імунізація Specific - active-passive immunization

Екстрена, неспецифічна (первинна хірургічна обробка рани), специфічна (активнопасивна імунізація) Emergency, non-specific (primary surgical treatment of the wound), specific (active-passive immunization)

Планова, шляхом введення правцевого анатоксину Plan, by introducing tetanus toxoid

Потерпілому ввести 3000 АО протиправцевої сироватки Inject 3000 AU of anti-tetanus serum to the victim

145 / 200
Жінка 60-ти років протягом останнього року стала відчувати слабкість, запаморочення, швидку втомлюваність. Останнім часом - задишка, парестезії. Об’єктивно: шкіра та слизові оболонки бліді з іктеричним відтінком. Сосочки язика згладжені. Печінка, селезінка у краю реберної дуги. У крові: Нb- 70 г/л, ер.- 1, 7 • 1012/л, КП- 1,2, макроцити. Призначення якого препарату є патогенетично обґрунтованим? A 60-year-old woman began to feel weakness, dizziness, quick fatigue over the past year. Recently, shortness of breath, paresthesias. Objectively: the skin and mucous membranes are pale with an icteric tinge. The papillae of the tongue are smoothed. Liver, spleen at the edge of the costal arch. In the blood: Hb- 70 g/l, er.- 1.7 • 1012/l, KP- 1.2, macrocytes. What drug is prescribed pathogenetically justified?

Вітамін В1 Vitamin B1

Аскорбінова кислота Ascorbic acid

Препарати заліза Iron preparations

Вітамін В6 Vitamin B6

Вітамін В12 Vitamin B12

146 / 200
Хворому 50 років, стаж роботи зварювальником 18 років, скаржиться на ядуху та сухий кашель. При рентгенобстеженні ОГП: у легенях на фоні підсиленого, сітчастодеформованого легеневого малюнку визначається велика кількість дрібних, округлих чітко окреслених тіней у середніх та нижніх відділах з обох боків, симетрично. Корені легень ущільнені, розширені. Який найбільш імовірний діагноз? The patient is 50 years old, has worked as a welder for 18 years, complains of wheezing and a dry cough. During the X-ray examination of the OGP: in the lungs against the background of an enhanced, mesh-deformed lung pattern, a large number of small, round, well-defined shadows in the middle and lower regions on both sides, symmetrically. The roots of the lungs are compacted, expanded. What is the most likely diagnosis?

Саркоїдоз Sarcoidosis

Міліарний карциноматоз Miliary carcinomatosis

Кардіогенний пневмосклероз Cardiogenic pneumosclerosis

Дисемінований туберкульоз легень Disseminated pulmonary tuberculosis

Пневмоконіоз Pneumoconiosis

147 / 200
Доношений новонароджений з масою тіла 4500 г народився в асфіксії з оцінкою за шкалою Апгар 4-6 балів. В пологах утруднене виведення плечового поясу. У неврологічному статусі загальномозкові розлади, виявлений тотальний верхній млявий парез - рука атонічна, пронована, не викликаються рефлекси - хапальний, Бабкіна, Моро. Укажіть сегменти ураження спинного мозку: A full-term newborn with a body weight of 4500 g was born asphyxiated with an Apgar score of 4-6 points. During delivery, the removal of the shoulder girdle is difficult. In the neurological status, general brain disorders, revealed total upper flaccid paresis - the hand is atonic, pronated, reflexes are not evoked - grasping, Babkina, Moro. Specify the segments of the spinal cord lesion:

СI - СII СИ - СII

СIII - СIV CIII - CIV

ТVI - ТVII TVI - TVII

ТI - ТV TI - TV

СV - ТI СВ - ТИ

148 / 200
Лікарями поліклініки проводиться статистичне дослідження результатів захворювань двох груп пацієнтів (тих, що знаходяться на диспансерному обліку і тих, що не знаходяться) в залежності від віку, статі, рівня гігієнічної культури. Який вид статистичних таблиць найдоцільніше використати для поглибленого аналізу залежності між наведеними ознаками? Polyclinic doctors conduct a statistical study of disease outcomes of two groups of patients (those who are on the dispensary register and those who are not) depending on age, gender, level hygienic culture. What type of statistical tables is the most expedient to use for an in-depth analysis of the dependence between the given characteristics?

Прості Simple

Комбінаційні Combination

Аналітичні Analytical

Розробні Under development

Групові Group

149 / 200
Жінка 57-ми років скаржиться на відчуття здавлення у стравоході, серцебиття, утруднення дихання при прийомі твердої їжі, іноді з’являється блювання повним ротом, вночі - симптом 'мокрої подушки'. Хворіє близько 6-ти місяців. Об’єктивно: температура - 36, 5°С, зріст - 168 см, вага -72 кг, Р8- 76/хв., АТ- 120/80 мм рт.ст. Рентгенологічно: стравохід значно розширений, у кардіальній частині - звужений. Яка патологія найбільш імовірно викликала дисфагію у хворої? A 57-year-old woman complains of a feeling of compression in the esophagus, palpitations, difficulty breathing when eating solid food, sometimes vomiting with a full mouth appears, at night - a symptom ' wet pillow'. He has been sick for about 6 months. Objectively: temperature - 36.5°С, height - 168 cm, weight -72 kg, R8 - 76/min., BP - 120/80 mm Hg. X-ray: the esophagus is significantly widened, in the cardiac part - narrowed. What pathology most likely caused the patient's dysphagia?

Грижа стравохідного отвору діафрагми Hernia of the esophageal opening of the diaphragm

Рефлюкс-езофагіт Reflux-esophagitis

Первинний езофагоспазм Primary esophageal spasm

Ахалазія кардії Achalasia cardia

Рак стравоходу Esophageal cancer

150 / 200
Жінка 35-ти років скаржиться на слабкість, часті випорожнення з домішками крові, зменшення ваги тіла на 8 кг за 6 місяців, періодично - підвищення температури до 37,8°С. Об’єктивно: t°- 37, 5°С, Р8- 86/хв., АТ- 110/70 мм рт.ст. Шкіра бліда, пальпаторно живіт помірно болісний. У крові: Нb-92 г/л, ШЗЕ- 35 мм/год. Колоноскопія: слизова прямої та сигмоподібної кишки зерниста, гіперемована, набрякла, нерівномірно потовщена (псевдополіпи), кровить, на її поверхні - ерозії та виразки. Яка патологія найбільш імовірно викликала ураження кишки у хворої? A 35-year-old woman complains of weakness, frequent bowel movements with blood impurities, a decrease in body weight by 8 kg in 6 months, periodically - an increase in temperature up to 37.8 °С. Objectively: t°- 37.5°С, Р8- 86/min., BP- 110/70 mm Hg. The skin is pale, the abdomen is moderately painful on palpation. In the blood: Hb-92 g/l , SZE- 35 mm/h. Colonoscopy: the mucous membrane of the rectum and sigmoid colon is granular, hyperemic, swollen, unevenly thickened (pseudopolyps), bleeding, erosions and ulcers on its surface. What pathology most likely caused the patient's bowel damage?

Виразковий коліт Ulcerative colitis

Хвороба Крона Crohn's disease

Синдром роздратованого кишечнику Irritable bowel syndrome

Рак прямої кишки Rectal cancer

Поліпоз кишки Intestinal polyposis

151 / 200
Юнак 20 -ти років звернувся до лікарні зі скаргою на біль, який виникає в нижній третині стегнової кістки під час навантаження та у спокої. Травму заперечує. Об’єктивно: шкіра звичайного кольору, пастозність і біль під час глибокої пальпації, зменшення рухів у колінному суглобі. На рентгенограмі дистального метаепіфізу стегна - зона деструкції та спікули. У крові: незрілі форми клітин, ознаки запалення відсутні. Який найбільш імовірний діагноз? A 20-year-old man went to the hospital complaining of pain in the lower third of the femur during exercise and at rest. He denies the injury. Objectively : normal-colored skin, pastiness and pain during deep palpation, decreased movements in the knee joint. On the X-ray of the distal metaepiphysis of the thigh, there is a zone of destruction and spicules. In the blood: immature forms of cells, signs of inflammation are absent. What is the most likely diagnosis?

Мармурова хвороба Marble disease

Гіперпаратиреоїдна дистрофія Hyperparathyroid dystrophy

Мієломна хвороба Myeloma

Остеогенна саркома Osteogenic sarcoma

Хронічний остеомієліт Chronic osteomyelitis

152 / 200
У чоловіка 47-ми років після підняття вантажу з’явилася різка біль у правій половині грудної клітини, почала наростати задуха, ціаноз шкірі та слизистої. Хворий 7 років тому хворів на інфільтративний туберкульоз правої легені. Останні три роки не обстежувався. При об’єктивному обстеженні перкуторно справа тимпаніт, при аускультації дихання різко послаблене. Яке ускладнення розвинулося у хворого? A 47-year-old man developed a sharp pain in the right half of the chest after lifting a load, suffocation began to increase, cyanosis of the skin and mucous membranes. Patient 7 years ago suffered from infiltrative tuberculosis of the right lung. He has not been examined for the past three years. On objective examination, percussive right tympanitis, on auscultation, breathing is sharply weakened. What complication developed in the patient?

Ексудативний плеврит Exudative pleurisy

Спонтанний пневмоторакс Spontaneous pneumothorax

Ателектаз правої легені Atelectasis of the right lung

Тромбоемболія легеневої артерії Thromboembolism of the pulmonary artery

Госпітальна пневмонія Nosocomial pneumonia

153 / 200
У хворої 50-ти років, яка страждає на гіпертонічну хворобу понад 10 років, на тлі стресу раптово підвищився артеріальний тиск до 200/110 мм рт.ст. Стан супроводжувався тремтінням тіла, головним болем, тахікардією, загальним збудженням, відчуттям жару та сухості в роті. Призначення яких препаратів є найбільш обґрунтованим? A 50-year-old patient who has been suffering from hypertension for more than 10 years had a sudden rise in blood pressure to 200/110 mmHg due to stress. Condition was accompanied by body tremors, headache, tachycardia, general excitement, a feeling of heat and dryness in the mouth. Which drugs are the most justified?

Інгібітори АПФ ACE inhibitors

Блокатори рецепторів ангіотензину II Angiotensin II receptor blockers

Антагоністи кальцію Calcium antagonists

Сечогінні Diuretics

β-адреноблокатори β-blockers

154 / 200
У хворої 64-х років на четвертий день перебування у терапевтичному стаціонарі з діагнозом: 'ІХС: Стенокардія напруги III ФК' виникло серцебиття і різко погіршилося самопочуття. Об’єктивно: тони серця глухі, аритмічні, систолічний шум на верхівці. ЧСС94/хв., дефіциту пульса немає. АТ-130/85 мм рт.ст. На ЕКГ: групові, політопні шлуночкові екстрасистоли, епізодами алоритмія. Розвиток якої фатальної клінічної ситуації можна передбачити? On the fourth day of her stay in a therapeutic hospital, a 64-year-old patient with a diagnosis of 'IHD: angina pectoris III FC' developed palpitations and her well-being deteriorated sharply. About' objectively: heart sounds are dull, arrhythmic, systolic murmur at the apex. HR94/min., no pulse deficiency. BP-130/85 mm Hg. On ECG: group, polytopic ventricular extrasystoles, episodes of allorhythmia. The development of which fatal clinical situation can predict?

Розвиток синдрому слабкості синусового вузла Development of sinus node weakness syndrome

Повна атріовентрикулярна блокада Complete atrioventricular block

Пароксизм миготливої аритмії Paroxysm of atrial fibrillation

Фібриляція шлуночків Ventricular fibrillation

Синоатріальна блокада Sinoatrial block

155 / 200
У жінки 46-ти років, що протягом 5-ти років страждає на гіпертонічну хворобу, розвинувся гіпертонічний криз. Скарги на серцебиття, відчуття пульсації у голові, ЧСС100/хв, АТ- 190/100 мм рт.ст. (гіперкинетичний тип гемодинаміки). Якому препарату слід надати перевагу? A 46-year-old woman, who has been suffering from hypertension for 5 years, developed a hypertensive crisis. Complaints of palpitations, a feeling of pulsation in the head, heart rate 100/ min, blood pressure - 190/100 mm Hg (hyperkinetic type of hemodynamics). Which drug should be preferred?

Інгібітор АПФ ACE inhibitor

а-адреноблокатор α-adrenoblocker

Дигідропіридиновий антагоніст кальцію Dihydropyridine calcium antagonist

Сечогінне Diuretic

β-адреноблокатор β-blocker

156 / 200
У пацієнта 54-х років м’яка первинна артеріальна гіпертензія, 1ХС: стенокардія напруги II ФК, СН II ст. Супутній діагноз: гастроезофагеальна рефлюксна хвороба, ерозивний езофагіт II стадія. Постійний прийом якого з препаратів може викликати у хворого посилення проявів гастроентерологічної патології? A 54-year-old patient has mild primary arterial hypertension, 1XS: angina pectoris II FC, HF II stage. Associated diagnosis: gastroesophageal reflux disease, erosive esophagitis Stage II. The constant intake of which of the drugs can cause the patient to increase the manifestations of gastroenterological pathology?

Омепразол Omeprazole

Ізосорбіду динітрат Isosorbide dinitrate

Гідрохлортіазид Hydrochlorothiazide

Метопролол Metoprolol

Еналаприлу малеат Enalapril maleate

157 / 200
Хворому на цукровий діабет I типу, якого доставили в реанімаційне відділення в стані кетоацидотичної коми, в перші три години в якості лікування внутрішньовенно введено: фізіологічний розчин 1,2 л, розчин Рінгера 400 мл, розчин калію хлориду 7,5% 200 мл, кокарбоксилаза 5 мл. Який ще засіб обов’язково має бути введений хворому? A patient with type I diabetes, who was brought to the intensive care unit in a state of ketoacidotic coma, in the first three hours was intravenously administered: physiological solution 1.2 liters , Ringer's solution 400 ml, potassium chloride solution 7.5% 200 ml, cocarboxylase 5 ml. What other means must be administered to the patient?

Плазма крові Blood plasma

Промедол Promedol

Еритроцитарна маса Erythrocyte mass

10% розчин кальцію хлориду 10% calcium chloride solution

4% розчин натрію гідрокарбонату 4% sodium bicarbonate solution

158 / 200
Хворий 17-ти років знаходиться на стаціонарному лікуванні з приводу гломерулонефриту. Скарги на виражені набряки по всьому тілі, зменшення кількості сечі, головний біль. У сечі: білок 7,1 г/л, лейк.- 1-2 у п/з, ер.- 3-4 у п/з. Білок у добовій сечі -3,8 г/л, діурез - 800 мл. Загальний білок 43,2 г/л, сечовина - 5,2 ммоль/л. Холестерин - 9,2 ммоль/л. Який з перерахованих синдромів гломерулонефриту найімовірніше має місце у хворого? A 17-year-old patient is undergoing inpatient treatment for glomerulonephritis. Complaints of severe swelling throughout the body, decreased urine output, headache. In the urine: protein 7 ,1 g/l, leuk.- 1-2 in p/z, er.- 3-4 in p/z. Protein in daily urine - 3.8 g/l, diuresis - 800 ml. Total protein 43.2 g/l, urea - 5.2 mmol/l. Cholesterol - 9.2 mmol/l. Which of the listed glomerulonephritis syndromes is most likely to occur in the patient?

Нефритичний Nephritic

Сечовий Urinary

Нефротичний Nephrotic

Гематуричний Hematuric

Змішаний Mixed

159 / 200
У 1-місячного хлопчика з симптомами збудження обвід голови складає 37 см, розміри великого тім’ячка 2x2 см. Дитина зригує після годування малими порціями молока; випорожнення нормальні за складом та об’ємом. М’язовий тонус у нормі. Який діагноз найбільш імовірний? A 1-month-old boy with symptoms of agitation has a head circumference of 37 cm, the size of the large parietal lobe is 2x2 cm. The child vomits after feeding small portions of milk; bowel movements are normal composition and volume. Muscle tone is normal. What is the most likely diagnosis?

Менінгіт Meningitis

Краніостеноз Craniostenosis

Мікроцефалія Microcephaly

Пілоростеноз Pylorostenosis

Пілороспазм Pilorospasm

160 / 200
На прийом до венеролога звернувся пацієнт, якому на підставі клінічних проявів і даних лабораторного обстеження був установлений діагноз трихомоніаз. При призначенні лікування цьому хворому основним препаратом варто вважати: A patient who was diagnosed with trichomoniasis on the basis of clinical manifestations and laboratory examination data came to see a venereologist. When treating this patient with the main drug, it should be considered:

Пімідель Pimidel

Інтерферон Interferon

Тінідазол Tinidazole

Тетрациклін Tetracycline

Бісептол Biseptol

161 / 200
Хворий 64-х років госпіталізований зі скаргами на задишку, відчуття важкості у правому підребер’ї, збільшення живота. Під час огляду лікар звернув увагу на відсутність набряків на ногах при явному асциті (ascitis praecox), набухання шийних вен, частий малий пульс. Під час аускультації серця визначається перикард-тон. ЕхоКГ: сепарація листків перикарда - 0,5 см, діаметр правого передсердя - 6 см. На рентгенограмі: серце нормальних розмірів, вздовж краю правих відділів серця визначається рентгеноконтрастний контур. Яке захворювання необхідно запідозрити в даному випадку? A 64-year-old patient was hospitalized with complaints of shortness of breath, a feeling of heaviness in the right hypochondrium, an enlarged abdomen. During the examination, the doctor noted the absence of swelling on the legs with obvious ascites (ascitis praecox), swelling of the neck veins, a frequent small pulse. During auscultation of the heart, the pericardial tone is determined. Echocardiogram: separation of the pericardial leaves - 0.5 cm, the diameter of the right atrium - 6 cm. On the radiograph: the heart is of normal size, along the edge of the right parts of the heart, a radiopaque contour is determined. What disease should be suspected in this case?

Констриктивний перикардит Constrictive pericarditis

Дилатаційну кардіоміопатію Dilated cardiomyopathy

Недостатність мітрального клапана Mitral valve insufficiency

Ексудативний перикардит Exudative pericarditis

Інфаркт міокарда в стадії утворення рубця Myocardial infarction in the stage of scar formation

162 / 200
Робітниця заводу з обробки шкіри, стаж роботи у контакті із урсолом 12 років, скаржиться на виражений свербіж шкіри. Об’єктивно: на кистях і пальцях рук, передпліччі, обличчі і шиї наявні симетричні папульозні поліморфні висипання. Стан погіршується після роботи, у вихідні дні та під час відпустки почуває себе краще. Що із переліченого слід застосувати у даному випадку? A worker at a leather processing plant, 12 years of work experience in contact with ursol, complains of severe itching of the skin. Objectively: on the hands and fingers, forearm, there are symmetrical papular polymorphic rashes on the face and neck. The condition worsens after work, on weekends and during vacation, he feels better. Which of the following should be used in this case?

Препарати сірки Sulphur preparations

Радіоактивні ізотопи Radioactive isotopes

Рентгенотерапія X-ray therapy

Дезінфікуючі розчини Disinfectant solutions

Антигістамінні препарати Antihistamines

163 / 200
Жінка 62-х років після підняття вантажу відчула гострий біль у поперековій зоні, сідниці, задньобоковій поверхні правого стегна, зовнішній поверхні правої гомілки та тильній поверхні стопи. Об’єктивно: слабкість переднього великогомілкового м’яза, довгого розгинача великого пальця, короткого розгинача пальців правої стопи. Знижений ахіловий рефлекс справа. Позитивний симптом Ласега. Який найбільш інформативний метод дослідження для уточнення діагнозу дискогенної компресії L5 корінця? After lifting a load, a 62-year-old woman felt sharp pain in the lumbar region, buttock, posterior surface of the right thigh, the outer surface of the right lower leg, and the back of the foot. objectively: weakness of the tibialis anterior muscle, extensor hallucis longus, extensor toes brevis of the right foot. Decreased Achilles reflex on the right. Positive Laseg sign. What is the most informative research method to clarify the diagnosis of discogenic compression of the L5 root?

Люмбальна пункція Lumbar puncture

Магнітно-резонансне сканування Magnetic resonance scan

Ангіографія Angiography

Електроміографія Electromyography

Рентгенографія хребта X-ray of the spine

164 / 200
Хвора 45-ти років доставлена машиною швидкої допомоги зі скаргами на різку загальну слабкість, нудоту, блювання, біль у животі. Останнім часом відзначає зниження апетиту, схуднення. Об’єктивно: гіперпігментація шкіри, АТ- 70/45 мм рт.ст., брадикардія. В додаткових дослідженнях знижений вміст альдостерону, кортизолу в крові, знижена екскреція 17-КС та 17-ОКС з сечею, гіпонатріємія, гіпохлоремія, гіпокаліємія. Які лікувальні заходи потрібно вжити? A 45-year-old patient was brought by ambulance with complaints of severe general weakness, nausea, vomiting, abdominal pain. Recently, she has noted a decrease in appetite, weight loss. About objectively: hyperpigmentation of the skin, blood pressure - 70/45 mm Hg, bradycardia. In additional studies, the content of aldosterone and cortisol in the blood is reduced, the excretion of 17-KS and 17-OKS in the urine is reduced, hyponatremia, hypochloremia, hypokalemia. measures should be taken?

Призначення інсуліну Prescription of insulin

Призначення глюкокортикоїдів, мінералокортикоїдів, дієти з підвищеним вмістом кухарської солі Prescription of glucocorticoids, mineralocorticoids, diet with a high content of table salt

Призначення альдостерону Aldosterone appointment

Призначення преднізолону Prednisone Prescription

Призначення дієтотерапії з підвищеним вмістом кухарської солі Prescription of dietary therapy with an increased content of cooking salt

165 / 200
До приймального відділення лікарні надійшов хворий з хриплим диханням (більш утруднений вдих), ціанозом шкіри, тахікардією та артеріальною гіпертензією. В анамнезі - бронхіальна астма. Годину тому робив інгаляцію сальбутамолу та забув зняти ковпачок, який аспірував при глибокому вдиху. Які дії лікаря? A patient with wheezing (more difficult breathing), cyanosis of the skin, tachycardia and arterial hypertension was admitted to the hospital's reception department. He has a history of bronchial asthma. An hour ago he was doing inhalation salbutamol and forgot to remove the cap, which aspirated during a deep breath. What actions did the doctor take?

Викликати анестезіолога та чекати на його появу Call an anesthesiologist and wait for his appearance

Ввести підшкірно дексаметазон Inject dexamethasone subcutaneously

Виконати прийом Геймліха Perform Heimlich maneuver

Одразу виконати конікотомію Perform conicotomy immediately

166 / 200
На санпропускник доставлено постраждалого зі скаргами на болі у ділянці тазу. Дві години тому під час вибуху був притиснутий автомобілем, що перевернувся. Стогне від болю. Об’єктивно: АТ- 70/40 мм рт.ст., ЧСС- 115/хв. Таз деформований. Вкорочення правої нижньої кінцівки. Органи черевної порожнини без патології. Оберіть оптимальний спосіб купірування больового синдрому: A casualty was brought to the hospital with complaints of pelvic pain. Two hours ago, during the explosion, he was crushed by an overturned car. He is moaning in pain. Objectively: Blood pressure - 70/40 mm Hg, heart rate - 115/min. The pelvis is deformed. Shortening of the right lower limb. The organs of the abdominal cavity are free of pathology. Choose the optimal method of stopping the pain syndrome:

Внутрішньокісткова анестезія у крило клубової кістки Intraosseous anesthesia in the wing of the iliac bone

Ненаркотичний анальгетик Non-narcotic analgesic

Внутрішньотазова анестезія Intrapelvic anesthesia

Провідникова анестезія Guide Anesthesia

Наркотичний анальгетик Narcotic analgesic

167 / 200
У хворої 37-ми років часті болісні сечовипускання, відчуття неповного спорожнення сечового міхура. Хворіє близько 15-ти років. У сечі: питома вага 1020, білок - 0,04 г/л, лейк.- 20-25 у п/з, ер.- 3-4 у п/з. При ультрасонографії - нирки без особливостей, об’єм сечового міхура 300 мл, його стінка потовщена до 0,5 см, трабекулярна у просвіті - ехозавис. Який метод додаткового дослідження необхідний цій пацієнтці у першу чергу для уточнення діагнозу? A 37-year-old patient has frequent painful urination, a feeling of incomplete emptying of the bladder. She has been sick for about 15 years. In urine: specific gravity 1020, protein - 0 ,04 g/l, leuk.- 20-25 in p/z, er.- 3-4 in p/z. During ultrasonography - kidneys without features, bladder volume 300 ml, its wall thickened to 0.5 cm, trabecular in the lumen - echo suspension. What method of additional research is necessary for this patient in the first place to clarify the diagnosis?

Екскреторна урографія Excretory urography

Цистоскопія Cystoscopy

Бакпосів сечі Urine backup

Оглядова урографія Review urography

Проба Нечипоренка Nechiporenko trial

168 / 200
Хворий 77-ми років скаржиться на неможливість помочитися, розпираючий біль над лоном. Захворів гостро 12 годин тому. Об’єктивно: над лоном пальпується наповнений сечовий міхур. Ректально: простата збільшена, щільно-еластична, з чіткими контурами, без вузлів. Міжчасткова борозенка виражена. При ультрасонографії - об’єм простати 120 см3, вона вдається в порожнину сечового міхура, паренхіма однорідна. Простатспецифічний антиген -5 нг/мл. Яке найбільш імовірне захворювання викликало гостру затримку сечі? A 77-year-old patient complains of the inability to urinate, throbbing pain above the pubic area. He became ill 12 hours ago. Objectively: a full bladder is palpated above the pubic area. Rectal : the prostate is enlarged, densely elastic, with clear contours, without knots. The interlobular groove is pronounced. At ultrasonography, the volume of the prostate is 120 cm3, it extends into the bladder cavity, the parenchyma is homogeneous. Prostate-specific antigen -5 ng/ml. What is the most likely did the disease cause acute urinary retention?

Рак простати Prostate cancer

Склероз простати Sclerosis of the prostate

Гострий простатит Acute prostatitis

Туберкульоз простати Tuberculosis of the prostate

Гіперплазія простати Prostate hyperplasia

169 / 200
У хлопчика 2-х років на другий день після профілактичного щеплення з’явилися підвищення температури до 38°С та біль у животі без певної локалізації. На 3-й день у дитини виникла червона папульозно-геморагічна висипка на розгинальних поверхнях кінцівок і навколо суглобів. Відмічається набряклість та незначна болючість колінних суглобів. З боку інших органів та систем - без патологічних змін. Який найбільш імовірний діагноз? A 2-year-old boy developed a temperature rise to 38°C and abdominal pain without specific localization on the second day after the preventive vaccination. On the 3rd day, the child developed a red papular-hemorrhagic rash on the extensor surfaces of the limbs and around the joints. There is swelling and slight pain in the knee joints. There are no pathological changes in other organs and systems. What is the most likely diagnosis?

Менінгококцемія Meningococcemia

Кропив’янка Hives

ДВЗ-синдром DVZ-syndrome

Геморагічний васкуліт Hemorrhagic vasculitis

Тромбоцитопенічна пурпура Thrombocytopenic purpura

170 / 200
Хворий 73-х років звернувся до лікаря зі скаргами на серцебиття, перебої в діяльності серця, ядуху з переважанням утрудненого вдиху. Р8- 96/хв., ЧСС- 128/хв., фібриляція передсердь. Тони серця ослаблені, систолічний шум на верхівці. Печінка +5 см. Набряки гомілок, ФВ- 51%. Для контролю частоти серцевих скорочень доцільно використати: A 73-year-old patient turned to the doctor with complaints of palpitations, interruptions in heart activity, shortness of breath with a predominance of difficult breathing. P8- 96/min., HR- 128/min., atrial fibrillation. Heart tones are weakened, systolic murmur at the apex. Liver +5 cm. Swelling of the lower legs, FV - 51%. To control the heart rate, it is advisable to use:

Фуросемід Furosemide

Триметазидин Trimetazidine

Амлодипін Amlodipine

Лізиноприл Lisinopril

Дигоксин Digoxin

171 / 200
Хворий 22-х років скаржиться на кашель з виділенням 'іржавого' харкотиння, епізоди затьмарення свідомості, підвищення температури тіла до 40,1°С, задишку, біль у м’язах. Об’єктивно: ЧД- 36/хв. Справа, нижче кута лопатки - тупий перкуторний звук, при аускультації - бронхіальне дихання. У крові: лейк.- 17,8 • 109/л, ШЗЕ- 39 мм/год. Який діагноз є найбільш імовірним? A 22-year-old patient complains of a cough with 'rusty' sputum, episodes of loss of consciousness, an increase in body temperature up to 40.1°C, shortness of breath, pain in muscles. Objectively: BH - 36/min. On the right, below the angle of the scapula - a dull percussion sound, during auscultation - bronchial breathing. In the blood: leuk. - 17.8 • 109/l, SZE - 39 mm/h . What diagnosis is the most probable?

Гострий бронхіт Acute bronchitis

Бронхоектатична хвороба Bronchoectatic disease

Негоспітальна пневмонія Community-acquired pneumonia

Туберкульоз легені Pulmonary tuberculosis

Рак легені Lung cancer

172 / 200
Для оцінки стану здоров’я населення складено та проаналізовано звіт про захворювання, що зареєстровані у населення району обслуговування (ф. 12). Який показник рахують на основі цього звіту? To assess the state of health of the population, a report on diseases registered among the population of the service area (f. 12) was compiled and analyzed. What indicator is calculated on the basis of this report ?

Показник загальної захворюваності Indicator of total morbidity

Показник госпіталізованої захворюваності Indicator of hospitalized morbidity

Показник основної неепідемічної захворюваності Indicator of the main non-epidemic morbidity

Показник захворюваності з тимчасовою втратою працездатності Incidence rate with temporary disability

Показник патологічної ураженості Indicator of pathological damage

173 / 200
У 12-ти річної дитини протягом 6-ти місяців періодично з’являється лихоманка до 38, 5°С, кашель, задишка, одноразово кровохаркання. БЦЖ - рубець відсутній. Дідусь хворіє на туберкульоз. В легенях розсіяні сухі і різнокаліберні вологі хрипи. Рентгенологічно: однотипна рівномірна дрібно-вогнищева інфільтрація легень, у верхніх частках тонкостінна каверна із слабковираженою перифокальною інфільтрацією. Яке захворювання імовірне? For 6 months, a 12-year-old child periodically develops fever up to 38.5°C, cough, shortness of breath, hemoptysis once. BCG - scar absent. Grandfather suffers from tuberculosis. Scattered dry and wet rales of various caliber are scattered in the lungs. X-ray: uniform small-focal infiltration of the lungs, in the upper lobes a thin-walled cavern with weak perifocal infiltration. What disease is likely?

Хронічний бронхіт Chronic bronchitis

Бронхіальна астма Bronchial asthma

Синдром Хамана-Річа Hamann-Rich syndrome

Туберкульоз легень Pulmonary tuberculosis

Вогнищева пневмонія Focal pneumonia

174 / 200
Хворий готується до операції з приводу варикозно розширених вен нижніх кінцівок. При огляді на підошвах виявлене мукоподібне лущення по ходу шкірних складок. Всі нігті стоп сірувато-жовтого кольору, потовщені і частково зруйновані. Про який дерматоз слід думати в даному випадку? The patient is being prepared for surgery for varicose veins of the lower extremities. On examination of the soles, mucoid peeling along the skin folds was detected. All toenails are grayish-yellow in color, thickened and partially destroyed. What dermatosis should be considered in this case?

Рубромікоз Rubromycosis

Мікроспорія Microsporia

Висівковий лишай Ban lichen

Мікробна екзема Microbial Eczema

Кандидоз Candida

175 / 200
У хворого після контакту з хімічними речовинами на виробництві раптово виникло стридорозне дихання. Осиплість голосу, 'гавкаючий' кашель, стала зростати задишка. Об’єктивно: акроціаноз. Який попередній діагноз? After contact with chemicals at work, the patient suddenly developed stridorous breathing. Hoarseness of voice, 'barking' cough, shortness of breath began to increase. Objectively: acrocyanosis. What previous diagnosis?

Набряк гортані Swelling of the larynx

Ателектаз легень Pulmonary atelectasis

Пневмоторакс Pneumothorax

Рак гортані Laryngeal cancer

ТЕЛА BODIES

176 / 200
Хворий 37-ми років госпіталізований до реанімаційного відділення у зв’язку з повторюваними кожні півгодини судомними тоніко-клонічними нападами. Між нападами до свідомості не приходить. АТ- 120/90 мм рт.ст., Ps- 100/хв. Вчора ввечері був на весіллі, вживав алкоголь. 5 років тому переніс закриту черепно-мозкову травму, забій головного мозку, після чого виникли поодинокі судомні напади з втратою свідомості. Хворий курс протиепілептичного лікування не проходив. Який препарат необхідно першочергово ввести для надання невідкладної допомоги? A 37-year-old patient is hospitalized in the intensive care unit due to repeated convulsive tonic-clonic seizures every half hour. He does not regain consciousness between seizures. AT-120 /90 mm Hg, Ps- 100/min. Was at a wedding last night, drank alcohol. 5 years ago suffered a closed brain injury, cerebral contusion, after which isolated seizures with loss of consciousness occurred. Patient course of antiepileptic treatment did not take place. What drug should be administered as a priority to provide emergency care?

Аміназин Aminazine

Сульфат магнію Magnesium sulfate

Тіопентал натрію Sodium Thiopental

Сибазон Sibazone

Оксибутират натрію Sodium oxybutyrate

177 / 200
Під час судово-медичного дослідження трупа судово-медичний експерт описав у тім’яноскроневій ділянці справа рану лінійної форми, розміром 6,4 см при зведених краях, краї нерівні, вкриті саднами; в глибині рани видно тканинні перетинки. Дайте назву описаної рани: During the forensic medical examination of the corpse, the forensic medical expert described a wound of a linear shape in the right parietotemporal area, the size of 6.4 cm with folded edges, the edges are uneven , covered with scabs; tissue membranes are visible in the depth of the wound. Give the name of the described wound:

Забита Bulk

Рубана Rubana

Розсічена Dissected

Різана Rizana

Колена Knees

178 / 200
Пацієнт 60-ти років скаржиться на практично постійне відчуття важкості і переповнення в епігастрії, що посилюється після їжі, відрижку з тухлим запахом, іноді блюванні з’їденою 1-2 дні тому їжею, схуднення. 12 років тому вперше виявлена виразка пілоричного каналу. Відзначав періодичні 'голодні' болі, з приводу яких приймав ранітидин. Погіршення протягом 3-х місяців. Об’єктивно: визначається 'шум плескоту' в епігастрії. Про яке ускладнення йдеться? A 60-year-old patient complains of an almost constant feeling of heaviness and fullness in the epigastrium, which worsens after eating, belching with a rotten smell, sometimes vomiting eaten 1- 2 days ago I eat, weight loss. Pyloric canal ulcer was first detected 12 years ago. He noted periodic 'hunger' pains, for which he took ranitidine. Worsening during 3 months. Objectively: a 'cluttering noise' is detected in the epigastrium. About which what is the complication?

Малігнізація виразки шлунка Malignancy of gastric ulcer

Стеноз пілоруса Stenosis of the pylorus

Пенетрація виразки шлунка Penetration of gastric ulcer

Стороннє тіло шлунка (безоар) Foreign body of the stomach (bezoar)

Функціональний спазм воротаря Functional spasm of the goalkeeper

179 / 200
Жінка 23-х років. Після перенесеного стресу з’явилися спрага, полідипсія, поліурія, схудла, наростала слабкість, потім з’явилися нудота, блювота, сонливість, знепритомніла. Госпіталізована. Глікемія - 28 ммоль/л, ацетон в сечі різко позитивний. Розпочато лікування кетоацидотичної коми. Коли доцільно розпочати профілактику гіпоглікемії шляхом введення 5% розчину глюкози? 23-year-old woman. After experiencing stress, thirst, polydipsia, polyuria appeared, she lost weight, weakness increased, then nausea, vomiting, drowsiness appeared, she fainted. She was hospitalized. Glycemia - 28 mmol/l, acetone in the urine is sharply positive. Treatment of ketoacidotic coma was started. When is it appropriate to start prevention of hypoglycemia by introducing a 5% glucose solution?

Після зниження рівня глікемії до 13-14 ммоль/л After lowering the blood sugar level to 13-14 mmol/l

При швидкості зниження рівня глікемії понад 5 ммоль/л за годину When the rate of lowering of blood glucose level is more than 5 mmol/l per hour

Після нормалізації рівня глікемії After normalization of blood glucose level

Після відновлення свідомості хворої After regaining consciousness of the patient

Через 2 години після початку інсулінотерапії In 2 hours after the start of insulin therapy

180 / 200
У електрозварювальника зі стажем роботи 15 років під час медичного огляду виявлено сухі хрипи в нижніх відділах легень. На рентгенограмі спостерігаються дифузні вузлики розміром 3-4 мм в середніх і нижніх відділах легень. Яке захворювання можна запідозрити? During a medical examination, an electric welder with 15 years of experience was found to have dry wheezing in the lower parts of the lungs. On the radiograph, diffuse nodules of 3-4 mm in size are observed in the middle and lower lung departments. What disease can be suspected?

Силікоз Silicosis

Карбоконіоз Carboconiosis

Бронхіт Bronchitis

Силікатоз Silicatosis

Металоконіоз Metaloconiosis

181 / 200
У підлітка 15-ти років при обстеженні в військкоматі виявлено інтервальний систолічний шум на верхівці серця, акцент II тону над легеневою артерією, тахікардія. Який із додаткових методів обстеження є найбільш інформативним для встановлення діагнозу? In a 15-year-old teenager, an interval systolic murmur at the apex of the heart, an accent of the II tone over the pulmonary artery, tachycardia was detected during examination at the military commissary. Which of the additional methods of examination is most informative for establishing a diagnosis?

Реографія Rheography

Рентгенографія X-ray

Ехокардіографія Echocardiography

Електрокардіографія Electrocardiography

Фонокардіографія Phonocardiography

182 / 200
Дівчинка 5-ти років скаржиться на болі в ділянці піхви, значні виділення гнійного характеру, які турбують протягом 5-ти днів і поступово посилюються. При огляді лікар виявив значний набряк зовнішніх статевих органів, почервоніння, гнійні виділення з піхви з неприємним запахом. При УЗД в ділянці піхви визначається ехо-позитивна тінь. Яка причина може призвести до такого стану у дітей? A 5-year-old girl complains of pain in the vagina, significant discharge of a purulent nature, which bothers her for 5 days and gradually increases. During the examination, the doctor found a significant swelling of the external genitalia, redness, purulent discharge from the vagina with an unpleasant smell. During ultrasound, an echo-positive shadow is determined in the vaginal area. What reason can lead to such a condition in children?

Пухлина піхви Vaginal tumor

Наявність стороннього тіла в піхві The presence of a foreign body in the vagina

Вульвовагініт Vulvovaginitis

Пухлина шийки матки Cervical tumor

Гнійний кольпіт Purulent colpitis

183 / 200
Постраждалого доставлено в тяжкому стані з місця ДТП. При обстеженні біль при осьовому навантаженні на крила таза та їх рухомість. В якому положенні слід іммобілізувати хворого? The victim was brought in a serious condition from the scene of the accident. During the examination, pain with axial load on the pelvic wings and their mobility. In what position should the patient be immobilized?

На боці з приведеними до живота стегнами On the side with the hips brought to the stomach

В положенні напівсидячи In a semi-sitting position

На спині з подушкою під тазом On the back with a pillow under the pelvis

На животі з подушкою під тазом On the stomach with a pillow under the pelvis

На спині в положенні за Волковичем On the back in the Volkovich position

184 / 200
Хвора 22-х років, зниженого харчування, вегетаріанка, звернулася в поліклініку зі скаргами на спотворення нюху, смаку, 'заїди' у кутах роту. Об’єктивно: виражена голубизна склер. Був встановлений діагноз: залізодефіцитна анемія. Який клінічний синдром має перевагу? A 22-year-old patient, a vegetarian, went to the polyclinic with complaints of distortion of smell, taste, and 'stuck' in the corners of the mouth. Objectively: pronounced bluish sclera. The diagnosis was established: iron-deficiency anemia. Which clinical syndrome is preferable?

Гемолітичний Hemolytic

Сидеропенічний Sideropenic

Іемологічний Iemological

Анемічний Anemic

Мієлодиспластичний Myelodysplastic

185 / 200
Пацієнт 50-ти років хворіє на гіпертонічну хворобу 20 років. Впродовж 2-х днів відмічає погіршення стану: головний біль, нудоту, запаморочення, набряк обличчя, задишку при незначних навантаженнях, які пов’язує з надмірним споживанням солоної їжі та перериванням курсу призначеної антигіпертензивної терапії. Об-но: АТ-180/120 мм рт.ст., ЧСС- 88/хв., ЧДР- 24/хв., послаблення дихання в заднє-базальних відділах легень. Надання допомоги пацієнту в стаціонарі слід розпочати з: A 50-year-old patient has been suffering from hypertension for 20 years. Over the course of 2 days, he notes a worsening of his condition: headache, nausea, dizziness, facial swelling, shortness of breath when minor stress, which is associated with excessive consumption of salty food and interruption of the course of antihypertensive therapy prescribed. in the posterior-basal parts of the lungs. Providing care to a patient in a hospital should begin with:

В/в введення лабеталолу IV labetalol injection

В/м введення 2 мл седуксену IV injection of 2 ml of seduxen

Сублінгвального прийому 10 мг корінфару Sublingual administration of 10 mg of corinfarum

В/в крапельного введення ізокету Isoket IV drip

В/в введення 80 мг фуросеміду IV administration of 80 mg furosemide

186 / 200
У хворого 37-ми років протягом тижня спостерігається різкий біль у лобній ділянці справа, погіршення дихання через ніс та слизово-гнійні виділення з правої половини. Об’єктивно: набряк слизової оболонки носа, слизово-гнійні виділення із середнього носового ходу. Для якого з перелічених захворювань найбільш характерні наведені симптоми? A 37-year-old patient has been experiencing sharp pain in the frontal area on the right for a week, worsening breathing through the nose, and muco-purulent discharge from the right side. Objectively: swelling of the nasal mucosa, muco-purulent discharge from the middle nasal passage. For which of the listed diseases are the given symptoms most characteristic?

Гемісинуїт Hemisinuit

Фронтит Frontite

Гайморит Sinusitis

Етмоїдит Ethmoiditis

Сфеноїдит Sphenoiditis

187 / 200
У населеному пункті планується будівництво багатопрофільної лікарні на 500 ліжок. Де на лікарняній ділянці необхідно розмістити поліклініку? The construction of a multidisciplinary hospital with 500 beds is planned in the settlement. Where on the hospital site should the polyclinic be located?

Розміщення поліклініки на території ділянки не дозволяється Placing a polyclinic on the site is not allowed

Біля господарської зони Near the economic zone

У центрі ділянки біля лікувальних корпусів In the center of the site near the medical buildings

У садово-парковій зоні In the park zone

Біля центрального входу Near the central entrance

188 / 200
Хвора 38-ми років скаржиться на 'при-пливи' відчуття жару, які повторюються до 5 разів на добу, головний біль у потиличній ділянці з підвищенням артеріального тиску, серцебиття, запаморочення, швидку втомлюваність, дратівливість, погіршення пам’яті. 6 місяців тому проведена операція в об’ємі екстирпації матки з придатками. Який найбільш імовірний діагноз? A 38-year-old patient complains of hot flushes that recur up to 5 times a day, a headache in the back of the head with increased blood pressure, palpitations, dizziness, rapid fatigue, irritability, memory impairment. 6 months ago, an operation was performed to extirpate the uterus with appendages. What is the most likely diagnosis?

Передменструальний синдром Premenstrual syndrome

Післякастраційний синдром Post-castration syndrome

Вторинна психогенна аменорея Secondary psychogenic amenorrhea

Фізіологічна пременопауза Physiological premenopause

Ранній патологічний клімакс Early pathological menopause

189 / 200
У хворої 28-ми років після стресу розвинулись: різка слабкість, запаморочення, потемніння в очах, нудота і втрата свідомості без судом. Об’єктивно: хвора непритомна, шкіра бліда, кінцівки холодні. Зіничні та сухожильні рефлекси збережені. АТ- 80/50 мм рт.ст., Ps- 102/хв., зниженого наповнення. Глюкоза крові - 5,7 ммоль/л. Який найбільш імовірний діагноз? After stress, a 28-year-old patient developed: sharp weakness, dizziness, darkening of the eyes, nausea and loss of consciousness without convulsions. Objectively: the patient is unconscious, the skin is pale, the extremities are cold. Pupillary and tendon reflexes are preserved. Blood pressure - 80/50 mm Hg, Ps - 102/min, reduced filling. Blood glucose - 5.7 mmol/l. What is the most likely diagnosis?

Істеричний припадок Hysterical attack

Гіпоглікемічна кома Hypoglycemic coma

Синкопальний стан Syncopal state

Транзиторне порушення мозкового кровообігу Transient disturbance of cerebral circulation

Епілептичний синдром Epileptic syndrome

190 / 200
Внаслідок вибуху цистерни з бензолом на хімічному заводі наявні загиблі та поранені у великій кількості (понад 50 осіб) з опіками, механічними травмами та отруєннями. Вкажіть основні елементи, які передбачає лікувально-евакуаційне забезпечення населення в цій ситуації: As a result of the explosion of a tank with benzene at a chemical plant, there are a large number of dead and injured (over 50 people) with burns, mechanical injuries and poisoning. Specify the main elements that provides medical and evacuation support for the population in this situation:

Ізоляція, рятування, відновлення Isolation, Rescue, Recovery

Сортування, евакуація, лікування Sort, evacuation, treatment

Надання медичної допомоги, евакуація, ізоляція Providing medical aid, evacuation, isolation

Сортування, відновлення, рятування Sort, Restore, Rescue

Сортування, надання медичної допомоги, евакуація Sort, medical assistance, evacuation

191 / 200
Хвора 25-ти років під час самообсте-ження виявила пухлину у верхньому зовнішньому квадранті правої молочної залози. При пальпації - безболісне, тверде, рухоме утворення молочної залози діаметром 2 см, периферичні лімфатичні вузли не змінені. При ультразвуковому дослідженні молочних залоз: у верхньому зовнішньому квадранті правої молочної залози об’ємне утворення підвищеної ехогенності, розміром 21х18 мм. Який найбільш імовірний діагноз? During a self-examination, a 25-year-old patient discovered a tumor in the upper outer quadrant of the right mammary gland. On palpation - a painless, firm, mobile mass of the mammary gland with a diameter of 2 cm, peripheral lymph nodes have not changed. During ultrasound examination of the mammary glands: in the upper outer quadrant of the right mammary gland there is a volumetric formation of increased echogenicity, measuring 21x18 mm. What is the most likely diagnosis?

Кіста молочної залози Breast cyst

Дифузна мастопатія Diffuse mastopathy

Мастит Mastitis

Рак молочної залози Breast cancer

Фіброаденома Fibroadenoma

192 / 200
Хвора 23-х років скаржиться на наявність пухлини у нижньому зовнішньому квадранті лівої молочної залози протягом 1-го року, що перед менструацією стає болісним і збільшується в розмірах. При пальпації: рухоме утворення, наповнене рідиною, до 3 см, із чіткими контурами, периферичні лімфатичні вузли не змінені. При ультразвуковому дослідженні молочних залоз: у нижньому зовнішньому квадранті лівої молочної залози об’ємне утворення зниженої ехогенності, розміром 31х29 мм. Який попередній діагноз? A 23-year-old patient complains of the presence of a tumor in the lower outer quadrant of the left mammary gland during the 1st year, which becomes painful and increases in size before menstruation. palpation: a mobile mass filled with liquid, up to 3 cm, with clear contours, peripheral lymph nodes are not changed. During ultrasound examination of the mammary glands: in the lower outer quadrant of the left mammary gland, a volumetric mass of reduced echogenicity, measuring 31x29 mm. What is the previous diagnosis?

Фіброаденома Fibroadenoma

Кіста молочної залози Breast gland cyst

Рак молочної залози Breast cancer

Фіброзна мастопатія Fibrous mastopathy

Масталгія Mastalgia

193 / 200
У хворого 18-ти років АТ- 120/70 мм рт.ст. В III міжребер’ї зліва від краю груднини - систолічне тремтіння. Ліва межа серця зміщена на 1 см назовні. У ІІ-ІІІ міжребер’-ях зліва систоло-діастолічний шум, який посилюється під час систоли, акцент ІІ тону. Який діагноз є найбільш імовірним? In an 18-year-old patient, blood pressure is 120/70 mm Hg. In the III intercostal space to the left of the edge of the sternum, there is a systolic tremor. The left border of the heart is displaced 1 cm outward. In the II-III intercostal space on the left, a systolic-diastolic noise that increases during systole, accent of the II tone. What diagnosis is the most likely?

Коарктація аорти Coarctation of the aorta

Відкрита артеріальна протока Open ductus arteriosus

Дефект міжшлуночкової перегородки Ventricular septal defect

Дефект міжпередсердної перегородки Atrial septal defect

- -

194 / 200
Через 2 тижні після пологів у породіллі з’явилися болі в молочній залозі, що наростали впродовж 3-х днів. Об’єктивно: температура тіла 39°С, мерзлякуватість, слабкість, гіперемія шкіри, збільшення, болючість та деформація молочної залози. При пальпації інфільтрату визначається ділянка розм’якшення та флюктуації. Який найбільш імовірний діагноз? 2 weeks after giving birth, the woman in labor developed pain in the mammary gland, which increased over the course of 3 days. Objectively: body temperature 39°С, frostbite, weakness, hyperemia of the skin, enlargement, tenderness and deformation of the mammary gland. During palpation of the infiltrate, the area of softening and fluctuation is determined. What is the most likely diagnosis?

Мастопатія Mastopathy

Інфільтративно-гнійний мастит Infiltrative-purulent mastitis

Серозний мастит Serous mastitis

Флегмонозний мастит Phlegmonous mastitis

Лактостаз Lactostasis

195 / 200
Дитині 3-х років у зв’язку із захворюванням на ГРВІ призначено: бісептол, парацетамол, назоферон. На третій день стан дитини погіршився: з’явилися біль у горлі, стоматит, кон’юнктивіт, гіперсалівація, болючі плями темно-червоного кольору на шиї, обличчі, грудях та кінцівках, потім на місті плям з’явились пухирі. Відмічалось враження слизових оболонок навколо рота та ануса. Який попередній діагноз? A 3-year-old child was prescribed: biseptol, paracetamol, nasoferon in connection with SARS. On the third day, the child's condition worsened: pain appeared in throat, stomatitis, conjunctivitis, hypersalivation, dark red painful spots on the neck, face, chest, and limbs, then blisters appeared on the spots. There was an impression of the mucous membranes around the mouth and anus. What was the previous diagnosis?

Бульозний дерматит Bullous dermatitis

Синдром Стівенса-Джонсона Stevens-Johnson syndrome

Сироваткова хвороба Serum sickness

Атопічний дерматит Atopic dermatitis

Вітряна віспа Chickenpox

196 / 200
Хвора 20-ти років скаржиться на відчуття нестачі повітря, тривалі ниючі болі в ділянці серця, дратівливість. Під час обстеження: загальний стан задовільний, лабільність пульсу, АТ-130/60 мм рт.ст. ЕКГ - порушення процесів реполяризації. У пацієнтки діагностована нейроциркуляторна дистонія за кардіальним типом. Вкажіть умови, в яких хвора повинна отримувати лікування: A 20-year-old patient complains of a feeling of lack of air, long-lasting aching pains in the heart area, irritability. During the examination: the general condition is satisfactory, pulse lability, BP- 130/60 mm Hg ECG - violation of repolarization processes. The patient is diagnosed with neurocirculatory dystonia of the cardiac type. Specify the conditions under which the patient should receive treatment:

Стаціонарне лікування в психіатричному відділенні Inpatient treatment in a psychiatric ward

Амбулаторне лікування Outpatient treatment

Стаціонарне лікування в терапевтичному відділенні Inpatient treatment in the therapeutic department

Стаціонарне лікування в кардіологічному відділенні Inpatient treatment in the cardiology department

Стаціонарне лікування в кардіохірургічному відділенні Inpatient treatment in the cardiac surgery department

197 / 200
Пацієнтка 20-ти років звернулась до лікаря жіночої консультації зі скаргами на відсутність менструацій протягом 7-ми місяців. З анамнезу: в ранньому віці хворіла на дитячі інфекції і ангіни, менархе з 13-ти років, місячні регулярні, менструальний цикл 28 днів, менструація триває 5-6 днів, безболісна. 7 місяців тому перенесла стрес. При гінекологічному огляді змін з боку матки і додатків не виявлено. Який найбільш імовірний діагноз? A 20-year-old patient turned to the doctor of the women's consultation with complaints about the absence of menstruation for 7 months. From the anamnesis: she suffered from childhood infections and sore throats at an early age , menarche since the age of 13, periods are regular, the menstrual cycle is 28 days, menstruation lasts 5-6 days, painless. 7 months ago, she experienced stress. During a gynecological examination, no changes were detected in the uterus and appendages. What is the most likely diagnosis?

Олігоменорея Oligomenorrhea

Вторинна аменорея Secondary amenorrhea

Первинна аменорея Primary amenorrhea

Несправжня аменорея False amenorrhea

Альгодисменорея Algodysmenorrhoea

198 / 200
У породіллі 24-х років післяпологовий період, 4-а доба. Пологи відбулися перші, своєчасні, без ускладнень. Загальний стан породіллі задовільний. Температура тіла 36,6oС Ps- 78/хв, ритмічний. Молочні залози в стані нагрубання. Дно матки на 2 см нижче пупка. Матка в нормотонусі, безболісна. Лохії кров’янисті, помірні. Випорожнення та сечовиділення в нормі. Яке лікування доцільно призначити? A 24-year-old woman in labor is in the postpartum period, the 4th day. The birth took place first, on time, without complications. The general condition of the woman in labor is satisfactory. Body temperature is 36.6oC Ps- 78/min, rhythmic. Mammary glands in a state of swelling. The bottom of the uterus is 2 cm below the navel. The uterus is normotonic, painless. Lochia is bloody, moderate. Defecation and urination are normal. What treatment should be prescribed?

Аналгетики Analgesics

Утеротонічні препарати Uterotonic drugs

Естрогенні гормони Estrogenic hormones

Антибіотики Antibiotics

Препарати, які припиняють лактацію Drugs that stop lactation

199 / 200
Хвору 47-ми років турбує пухлиноподібний утвір на передній поверхні шиї в ділянці щитоподібної залози. Відмічає прогресивне збільшення утвору. Об’єктивно: в правій частці щитоподібної залози відмічається утвір діаметром близько 4 см з гладенькою поверхнею, рухомий, підвищеної щільності. Незначно виражені симптоми тиреотоксикозу. Який із додаткових методів обстеження є найбільш інформативним для уточнення діагнозу? A 47-year-old patient is concerned about a tumor-like formation on the front surface of the neck in the area of the thyroid gland. He notes the progressive increase of the formation. Objectively: a formation is noted in the right lobe of the thyroid gland with a diameter of about 4 cm with a smooth surface, mobile, of increased density. Mildly expressed symptoms of thyrotoxicosis. Which of the additional examination methods is the most informative for clarifying the diagnosis?

Тонкоголкова пункційна біопсія Fine needle puncture biopsy

Рентгенографія шиї X-ray of the neck

УЗД Ultrasound

Визначення тиреоїдних гормонів Determination of thyroid hormones

Визначення білковозв’язанного йоду Determination of protein-bound iodine

200 / 200
У структурі населення регіону питома вага осіб віком від 0 до 14 років - 25%, питома вага осіб від 50 років і старше - 30%. Яке поняття найбільш точно характеризує цю демографічну ситуацію? In the structure of the population of the region, the specific weight of people aged 0 to 14 years is 25%, the specific weight of people 50 years and older is 30%. Which concept is the most accurate characterizes this demographic situation?

Когортний тип населення Cohort population type

Прогресивний тип вікової структури населення Progressive type of population age structure

Регресивний тип вікової структури населення Regressive type of population age structure

Стаціонарний тип вікової структури населення Stationary type of population age structure

Імміграція населення Population immigration